Examen cirugia

January 3, 2018 | Author: azrael_fire45 | Category: Prostate Cancer, Diseases And Disorders, Medicine, Clinical Medicine, Medical Specialties
Share Embed Donate


Short Description

Download Examen cirugia...

Description

Universidad La Salle. Facultad Mexicana de Medicina. Curso de Extensión Universitaria para la Preparación del Examen Nacional para Aspirantes a Residencias Médicas. Examen Módulo de Cirugía 14 de julio 2010. Modalidad a Distancia.

1.- La siguiente ametropía está causada por una diferencia de poder entre el eje horizontal y el eje vertical de la cornea:

a) b) c) d)

Miopía. Queratocono. Hipermetropía. Astigmatismo.

En el astigmatismo los errores de refracción se deben a la diferencia que existe entre el poder de refracción del eje horizontal con el vertical.

Stephen, et al. Current Medical Diagnosis and Treatment. McGraw Hill, 2008, pgs 141.

2.- Se trata de paciente masculino de 67 años, con antecedentes de constipación y dolor abdominal en fosa ilíaca izquierda recurrente. El enema baritado muestra múltiples divertículos en el sigmoides. ¿Cuál es el tratamiento más apropiado? a) Refección quirúrgica.

b) Colonoscopía con biopsia. c) Ranitidina. d) Dieta alta en residuos.

Braunwald E, fauci A, Kasper D, Hauser S, Longo D, jameson L. Harrison Principios de Medicina Interna. 15° edición. Mc Graw Hill 2001. 1982.

Algunos pacientes presentan cuadros recurrentes de dolor en fosa ilíaca izquierda sin signos clínicos ni patológicos de diverticulitis aguda. Presentan síntomas alternativos de estreñimiento y dolor, que se alivia mediante la defecación y eliminación de gases. En el enema opaco se observan divertículos típicos sin indicios de inflamación o estenosis. En algunos pacientes, el dolor es tan intenso que justifica su hospitalización y el ayuno, dado que los alimentos agravan el dolor al producir contracción colónica. El tratamiento necesario es la administración de anticolinérgicos, que disminuyen el peristaltismo. Tras la recuperación, debe iniciar una dieta alta en residuos o recibir un laxante de tipo inerte como la hemicelulosa, salvado no procesado o extracto de psilio. La extirpación quirúrgica no está indicada amenos que se produzca diverticulitis aguda.

3.- Acción que se debe realizar en un paciente masculino de 52 años con titulo de antígeno prostático de 72 ng/dl :

a) b) c) d)

Prostatectomia radical. Resección transuretral de próstata. RM pelvis. Biopsia con aguja guiada por USG.

La forma ideal para diagnosticar el cáncer de próstata es realizar una biopsia transrectal de próstata. Las dos principales indicaciones para realizar la biopsia de próstata son un tacto rectal anormal o un nivel de antígeno prostático elevado. Es decir que si usted tiene un tacto rectal normal pero su nivel de antígeno es alto, puede necesitar una biopsia de próstata para detectar cáncer de próstata. Más aún, usted puede tener un nivel de antígeno normal, pero si a juicio de su urólogo, el tacto rectal es anormal, puede igualmente requerir una biopsia de próstata. 1. 2. 3. 4. 5. 6.

Sólo el 1% de la glándula es examinada por biopsia. Eso significa que la posibilidad de no detectar cáncer presente es del 99% . Si el cáncer es muy pequeño seguramente no será detectado. Si el cáncer está localizado en un punto de difícil acceso probablemente no será detectado. Si el examen patológico es muy superficial o rápido, seguramente no será detectado El cáncer de próstata es de consistencia dura y es muy fácil que la dureza del tejido desvíe la aguja, especialmente con la técnica de aguja flexible. Cuando se reutilizan agujas estas pierden filo y por tanto no cortan bien. Ello hace más dolorosa la biopsia (son entre 7 y 24 disparos) y se reduce la cantidad de tejido adecuado para el examen patológico. Y la reutilización de las pistolas reduce la respuesta de sus resortes, haciendo el corte también menos efectivo y más doloroso. Cuanto más se usa un resorte, más se gasta.

Antes de practicarse la biopsia de próstata el paciente debe firmar un consentimiento escrito en el cual se le debe informar acerca de los riesgos de la intervención, así como de la posibilidad que existe de no detectar cáncer pese a estar este presente. La lista de las complicaciones aparece a continuación: Complicaciones de la biopsia de próstata: a- Septicemia: Es una infección severa de la sangre que se agrava rápidamente hasta originar shock séptico que es potencialmente mortal. Suele causar insuficiencia suprarrenal, coagulación intravascular diseminada, complicaciones renales, abdominales, pulmonares, óseas y neurológicas. b- Infecciones trasmitidas como hepatitis y SIDA así como abscesos en la zona puncionada e infecciones urinarias. c- Hemorragia y hematomas, sangre en la orina, sangrado abundante por el ano y en el semen. d- Fístulas arteriovenosas. e- Siembras del tumor pues se liberan células cancerosas que viajan por la sangre. f- Perforación de la vejiga.

g- Retención de orina, originada por la inflamación que se origina en la próstata tras la biopsia. h- Severo dolor. i- Accidente cerebrovascular (supimos de el pues una de las personas qué nos consultó por email no siguió nuestro consejo y se practicó la biopsia. j- Absceso perineal, que puede aparecer tardíamente (2 meses o más) k- Disfunción eréctil. Las relaciones sexuales se deterioran o dificultan en grados variables. l- Reducción del deseo y actividad sexual que puede ser permanente.

Referencias: Ries LAG, Eisner MP, Kosary CL, et al. (eds). SEER Cancer Statistics Review, 1975–2001, National Cancer Institute. Bethesda, MD, 2004 (http://seer.cancer.gov/csr/1975_2001). Thompson IM, Pauler DK, Goodman PJ, et al. Prevalence of prostate cancer among men with a prostate-specific antigen level 4.0 ng per milliliter. The New England Journal of Medicine 2004; 350(22):2239–2246.

Keetch DW, Catalona WJ, Smith DS. Serial prostatic biopsies in men with persistently elevated serum prostate specific antigen values. The Journal of Urology 1994; 151(6):1571– 1574.

4.- Femenino de 64 años que cursa con retinopatía diabética, en este momento presenta hemorragias subhialoideas, usted determina que ésta paciente se encuentra en el siguiente estadío:

a) b) c) d)

Retinopatía diabética no proliferativa moderada. Retinopatía diabética no proliferativa severa. Retinopatía diabética con edema macular. Retinopatía diabética proliferativa.

Clasificación: La Retinopatía Diabética se puede clasificar desde etapas más iniciales o leves, a otras más avanzadas o graves de acuerdo a su aspecto oftalmoscópico, es decir, de acuerdo al aspecto que se pueda apreciar en el examen de Fondo de Ojo. Para realizar este examen es de cardinal importancia dilatar al paciente. Según el ETDRS la Retinopatía Diabética se puede clasificar en una etapa temprana o Retinopatía Diabética No Proliferativa (RDNP) y una más avanzada o Retinopatía Diabética Proliferativa (RDP). La RDNP se subdivide a su vez en leve, moderada, severa y muy severa. La RDP se subdivide en temprana, de alto riesgo y avanzada. El Edema Macular es un evento que puede suceder en cualquier momento de la progresión de la Retinopatía Diabética.

Retinopatía Diabética No Proliferativa (RDNP) Los cambios que se producen en la RDNP están limitados a la retina y no pasan más allá de la membrana limitante interna de esta. Los elementos característicos que se pueden apreciar en el examen oftalmoscópico comprenden microaneurismas, hemorragias intraretinales en forma de manchas (dot-and-blot), edema retinal, exudados céreos o lipídicos (hard exudates), dilataciones venosas y rosarios venosos (venous beading), anormalidades intraretinales microvasculares (IRMA), manchas algodonosas (cotton-wool o soft), anormalidades arteriolares y áreas de cierre capilar. De estas alteraciones, las hemorragias intraretinales, los exudados céreos, las manchas algodonosas y las dilataciones venosas, pueden ser vistas por el médico internista o médicos no oftalmólogos, usando un oftalmoscopio directo y, como ya dijimos, con dilatación pupilar.

No detallaremos las características oftalmoscópicas específicas de cada una de las subdivisiones de la RDNP. Sin embargo, es importante mencionar que las alteraciones más importantes son: las hemorragias intraretinales, las dilataciones venosas y las anormalidades intraretinales microvasculares. A mayor número estas, la RDNP aumenta en su severidad y empeora en su pronóstico. Según el ETDRS, los pacientes con RDNP severa tienen un 15% de posibilidades de progresar a RDP de alto riesgo en un año y los que padecen RDNP muy severa tienen un 45% de posibilidades de progresar a RDP de alto riesgo en un año. Retinopatía Diabética Proliferativa (RDP) La isquemia progresiva que se produce en la Retinopatía Diabética, debido al cierre capilar, tiene como consecuencia la formación de vasos retinales de neoformación o Neovasos, los cuales junto a un tejido fibroso que los acompaña, proliferan más allá de la retina. Es lo que se denomina proliferación extraretinal. Estos neovasos son histológicamente diferentes a los vasos retinales normales. No conservan la barrera hematoretinal, sangran con mayor facilidad, y como mencionamos recientemente, crecen sustentados en un tejido fibroso, el que tiene capacidad contráctil. Estas características recién mencionadas, son las responsables de la Filtración Extravascular, las Hemorragias Preretinales o Vítreas y los Desprendimientos de Retina Traccionales, respectivamente. En etapas avanzadas los neovasos pueden crecen en la superficie iridiana y en el ángulo iridocorneal. El crecimiento del tejido fibrovascular sobre el ángulo, lo hace impermeable a la salida de humor acuoso, originando aumento de la presión intraocular y glaucoma secundario. Este glaucoma se conoce como Glaucoma Neovascular, el cual es de muy mal pronóstico. La aparición de estos neovasos es lo que define a la Retinopatía Diabética Proliferativa. Los neovasos se observan con mayor frecuencia en el nervio óptico o cercanos a las arcadas vasculares, pero se pueden encontrar en cualquier parte del fondo de ojo. Es posible ver los neovasos usando un oftalmoscopio directo y dilatación pupilar. La RDP evoluciona en tres etapas de menor a mayor severidad: temprana, de alto riesgo y avanzada. Esto está dado por la ubicación y extensión de los neovasos, la presencia o ausencia de hemorragia vítrea y la presencia o ausencia de desprendimiento de retina con compromiso foveal. Bibliografía 1. AAO, Basic and Clinical Sciences Course, Retina 2000-2001 2. Verdaguer TJ, Retinopatía Diabética. Clasificación, Normas para Pesquisa y Tratamiento 3. Freeman WR, Practical Atlas of Retinal Disease and Therapy, Second Edition 4. Alvarez NR, Diploma de Actualización en Medicina 1999, Modulo de Diabetes, Diagnostico y Tratamiento de la Retinopatía Diabética.

5.- El diagnóstico mas probable en un paciente femenino de 23 años que presenta en una radiografía con datos de tumoración multiloculada en la zona epifisiaria del extremo distal de fémur izquierdo, es el siguiente: a) Sarcoma osteogénico. b) Tumor de células gigantes. c) Sarcoma de Ewing. d) Mieloma múltiple.

INTRODUCCIÓN: El tumor de células gigantes es un tumor raro, constituye el 5% de los tumores óseos primarios (1). Ocurre en pacientes entre los 20 y 40 años, siendo raro en los menores de 10 y en mayores de 50 años. Tiene un ligero predominio del sexo femenino y es más frecuente en países orientales que occidentales (2). Habitualmente se localizan en huesos largos, con afectación de la epífisis y de localización excéntrica, pueden propagarse a metáfisis, provocar destrucción cortical y extensión eventual a tejidos blandos y al espacio articular. Los tres sitios más habituales de localización (3), por orden de frecuencia, son: 1º Extremo distal del fémur 2º Extremo proximal de tibia 3º Extremo distal del radio. Clínicamente, el dolor es el signo más frecuente de presentación, tanto si se asocia o no a fractura patológica (4). Localmente la piel puede estar hiperérmica, eritematosa y si el tumor crece puede aparecer circulación colateral con presencia de masa palpable. La clínica sistémica es infrecuente y si el TCG tiene localización raquídea o sacra presenta signos y síntomas neurológicos (5). El diagnóstico del TCG suele realizarse por las manifestaciones clínicas y radiográficas (tabla I), teniendo siempre en cuenta los posibles diagnósticos diferenciales (tabla II). Como pruebas complementarias, la RMN es el método más provechoso para determinar la extensión y el estadiaje, la Gammagrafía es utilizada para detectar TCG multicéntricos que son raros (< 1%) (6) y los parámetros de laboratorio suelen ser normales.

Tabla I: Características radiológicas del TCG Lesión osteolítica, excéntrica y epifisaria. Bordes generalmente bien definidos pero no esclerosos. Afectación parcial metafisaria. Expansión a hueso subcondral, adelgazamiento y abombamiento cortical (ruptura) y expansión a tejidos blandos. Minima o ninguna trabeculación.

Pals, S.D., and Wilkins, R.M.:Giant cell tumor of bone treated by curettage, cementation, and bone grafting. Orthopedics.1992. 15: 703-708.

6.- Los siguientes son síndromes epilépticos:

a) Sx. Down, Sx. Klinefelter. b) Sx. Apert, Sx Cry Duchant. c) Sx de Rasmussen, Sx de West. d) Sx. Marfan, Sx Elers Danlos.

Síndrome de Rasmussen: Descripción. El Síndrome de Rasmussen epilepsia infantil y pertenece a las "epilepsias catastróficas", por lo general aparece entre los 3 y 12 años, caracterizada por crisis focales o parciales incontrolables, hemiparesia y deterioro intelectual cognitivo. Síntomas: Las crisis más típicas son la epilepsia parcial contínua que es el movimiento no controlado de una mano o un pie durante días, semanas o meses, es decir, una crisis focal permanente. Estos niños suelen presentar además muchos otros tipos de crisis focales y convulsiones. Crisis rebeldes a todos los tratamientos que ocurren a diario (con frecuencia muchas al día). Aparición de una debilidad progresiva de un lado del cuerpo que lleva en meses o años a la parálisis de un brazo y una pierna. Problemas de comportamiento y del aprendizaje. Con frecuencia esta encefalitis lleva a una disminución del coeficiente

intelectual. Es una enfermedad terrible y devastadora y es importante el diagnóstico temprano ya que la cirugía de la epilepsia es curativa en estos casos.

Síndrome de West (espasmos infantiles) El síndrome de West es una epilepsia que aparece en los niños pequeños (lactantes) y se diagnostica por tres características: Los niños presentan espasmos de las brazos y tronco (espasmos infantiles) Se produce en la mayoría de ellos una regresión intelectual y del desarrollo Motor. En el electroencefalograma aparece un patrón típico (llamado hipsarritmia) El síndrome de West puede producirse en relación con lesiones graves cerebrales (por ejemplo, las malformaciones cerebrales de la Esclerosis tuberosa) o puede no tener un origen aclarado (síndrome de West criptogénico).

Los niños con espasmos infantiles tienen un difícil pronóstico, ya que alguno de ellos, pueden presentar crisis no controladas durante largo tiempo y un retraso mental. Bibliografía: 1. Arroyo S, Campistol J, Comes E, Fossas P, Martínez I, Padró Ll et al. El tratamiento de las epilepsias. Guía terapeútica de la Societat Catalana de Neurologia. Rev Neurol 1999; 29:754-766. 2. Arroyo S. Surgery for epilepsy: evaluation and indication. Neurologia 1996; 11 (Suppl 4):122-128. 3. Arroyo S. Diagnosis and treatment of drug-resistant epilepsy. Neurologia 1996; 11:56-68. 4. Pardo C, Vining EPG, Arroyo S, Freeman JM. Cytokines are involved in the immunopathological mechanisms of neuronal injury in Rasmussen´s syndrome. Epilepsia 1995; 36 (Suppl. 4):30. (Abstract) 5. Arroyo S, Freeman JM. Epilepsy surgery in children: state of the art. Adv Pediatr 1994; 41:53-81.

7.- Al realizar ultrasonografía renal, usted observa una masa bien delimitada, en el que se interrumpe el contorno renal, sin presencia de ecos en su interior y refuerzo posterior, ¿Qué diagnóstico es el más probable?

a) Absceso renal.

b) Hipertrofia de columna de Bertin. c) Carcinoma renal. d) Quiste renal simple.

Los criterios para el diagnóstico ultrasonográfico de un quiste simple renal son los siguientes: 1.- Forma ovoide o esférica. 2.-Ausencia de ecos internos (quiste anecoico). 3.-Presencia de una pared delgada y lisa bien definida que lo separa del parénquima. 4.-.El refuerzo acústico más allá de la pared posterior del quiste es proporcional a su contenido líquido. 5.-Se observa una banda estrecha de forma acústica por fuera del borde externo. Si se cumplen con todos estos criterios no es necesario realizar más estudios para diagnosticar la masa. Sin embargo, ocasionalmente la ecografía puede no resultar la técnica más idónea (por ejemplo cuando hay calcificación de la pared del quiste, éste es hemorrágico o existen múltiples quistes).

Bibliografía recomendada

-Brenner and rector The Kidney 2 volúmenes 2004 Saunders 2525 pag.

8.- Una de las siguientes hipoacusias es característica de la enfermedad de Meniere: a) Hipoacusia neurosensorial fluctuante. b) Hipoacusia neurosensorial sin reclutamiento. c) Hipoacusia de transmisión. d) Hipoacusia mixta, de transmisión y neurosensorial.

La enfermedad de Meniere, tal vez la más sobrediagnosticada causa de vértigo, es una entidad clínica específica, que cursa en general con: -Hipoacusia neurosensorial fluctuante y progresiva. -Crisis paroxisticas y episódicas de vértigos. -Acufenos. Muchas veces estas crisis están precedidas de "auras" como: -Sensación de plenitud en oído. -Intensificación del acufeno. El comienzo de la crisis es en general agudo e intenso; puede durar varias horas y excepcionalmente más de 2 días. Otra característica son los periodos asintomáticos de varias semanas y/o meses. En la Enfermedad de Meniere persistente, la frecuencia de los ataques aumenta con el tiempo, volviéndose cada vez más breves y menos severos y con menos auras. Si bien la etiología de la E. de Meniere es incierta, el cambio patológico en el Oído interno es la Hidropesia endolinfatica, que se produciría por: -Hiperproducción de endolinfa de características bioquímicas normales (rica en K y sin proteínas como la normal) por aumento de presión del segmento arterial de la Estría Vascular -Reabsorción insuficiente por deficiencia del Saco Endolinfatico (embriogenica - genetica infecciosa-traumática) Esta alteración en la homeostasia endolinfa-perilinfa, determina la hidropesía con:

• • •

Hipertensión en la endolinfa que comprime el Órgano de Corti (sensación de plenitud, disacusia y reclutamiento) Disminución del potencial endococlear por alteración de los canales de los cationes K y Ca.(hipoacusia) Ruptura de la M de Reissner, permitiendo el pasaje de Na a la endolinfa y de K a la perilinfa , lo que causa la desvalorización del polo apical de las células ciliadas y la irritación de las terminales nerviosas perilinfáticas del vestibular.(crisis vertiginosas)

Esta sintomatología puede estar asociada con otros trastornos sistémicos subyacentes como: •

Estados de retención de líquidos. -Hiperlipidemias. -Hipotiroidismo. -Sífilis congénita o latente. -Diabetes. -Alergias.

Bibliografía: • • • • • • •

British National Formulary. 53rd ed. London: British Medical Association and Royal Pharmaceutical Society of Great Britain; 2007. Boyev, KP. Meniere's disease or migraine? The clinical significance of fluctuating hearing loss with vertigo. Arch Otolaryngol Head Neck Surg 2005; 131:457-9 Brenner M, Hoistad D, Hain TC. Prevalence of thyroid dysfunction in Meniere's Disease. Arch Otolaryngol Head Neck Surg 2004; 130: 22 Celestino D, Ralli G. Incidence of Meniere’s disease in Italy. Am J Otology 1991; 12; 135-8. Chia SH, Gamst AC, Anderson JP, Harris JP. Intratympanic gentamicin therapy for Meniere's disease: a meta-analysis. Otol Neurotol 2004; 25:544. CKS. Meniere disease whole view. NHS; 2008 [acceso 28/2/2008]. Cohen-Kerem R, Kisilevsky V, Einarson TR, Kozer E, Koren G, Rutka JA . Intratympanic gentamicin for Meniere's disease: a meta-analysis. Laryngoscope 2004; 114:2085.

9.- Se trata de paciente femenino de 30 años de edad la cual sufre caída sobre su hombro izquierdo al conducir su bicicleta en una competencia. En la exploración presenta

deformidad y signo de la “tecla de piano” en la extremidad distal de la clavícula. ¿Cuál es el diagnóstico más probable de de la lesión?

a) Luxación acromio-clavicular. b) Fractura de la cabeza humeral. c) Luxación escápulo-humeral. d) Fractura de escápula.

La luxación acromio-clavicular es una de las lesiones traumáticas del hombro que día a día se hace más común, en virtud al aumento de la frecuencia de accidentes de tránsito y deportivos, evidente en la última década. El aporte funcional de la articulación acromio clavicular a la biomecánica del hombro, fuera de contribuir con el 20% del total de los arcos de movimiento, y quizás más importante, es el de actuar como fulcro, sobre el cual se completan la abducción y flexión del hombro, importantísimas para el desempeño armónico y funcional de la extremidad superior.

La mayoría de los pacientes sufre la lesión en actividades deportivas y accidentes automovilísticos, y dependiendo de la duración e intensidad de las fuerzas aplicadas, se producirán las lesiones descritas como G I, G II y G III de Allman1: • G I: Lesión intraarticular sin ruptura del complejo capsuloligamentoso acromioclavicular, generalmente producida por impacto directo de mediana intensidad sobre la cara externa del acromión. • G II: Subluxación de la clavícula por ruptura de la cápsula y de los ligamentos acromioclaviculares, por una fuerza aplicada en dirección superior y lateral, produciendo inicialmente una rotación externa de la escápula que toma como punto de pivote los ligamentos coracoclaviculares. • G III: Luxación completa de la clavícula en dirección postero superior por ruptura de los ligamentos coracoclaviculares, debido a la progresión de la fuerza lesionante descrita en el G II. Generalmente se encuentran desgarros de los músculos deltoides y trapecio en sus inserciones a nivel de clavícula distal y acromión.

DIAGNÓSTICO CLÍNICO A. Mecanismo de trauma. B. Cuadro clínico. II. Signos de trauma agudo (escoriación equinosis etc. aspecto postero lateral del hombro). II. Dolor. III. Limitación funcional. IV. Deformidad (signo de la tecla y signo de la charretera)

DIAGNÓSTICO RADIOLÓGICO: Se estableció como técnica estándar en este estudio, de acuerdo a lo indicado en la literatura habitualmente (L. Böhler 1957 citado por otros, 4), la toma de placas AP de la articulación acromio-clavicular (es necesario especificarlo así, pues rutinariamente en las placas simples de hombro, el tiempo de exposición mayor hace que la articulación acromioclavicular se vea radiolúcida, (Zariczny)

1. Weaver, J.K., Dunn, H.: Treatment of acromioclavicular injuries, especially complete acromioclavicular separation. J. Bone and Joint Surg., 54-A; 1.187-1.194.

10.- Ingresa al servicio de Traumatología masculino de 35 años postraumatizado por colisión automovilística con cuadro de pérdida progresiva de fuerza en miembros inferiores, los reflejos osteotendinosos están abolidos. Al realizarse radiografías simples muestran una fractura por compresión de L1 con desplazamiento del muro posterior y

acuñamiento anterior de un 50%. ¿Para valorar la ocupación del canal raquídeo, cuál de las siguientes pruebas indicaría?

a) Una gammagrafía ósea. b) Una tomografía cervical. c) Rx ap y lateral con foco en L5. d) Una TAC vertebral centrada en región dorso lumbar.

Tomografía computarizada (TC) En líneas generales, podemos decir que sus indicaciones (con carácter urgente) en los TRM, son todas aquellas lesiones detectadas o sospechadas en las radiografías simples, que puedan suponer riesgo de lesión medular por desplazamientos ulteriores (lesiones inestables), o que ya estén produciendo daño neurológico susceptible de mejorar o estabilizarse tras descompresión quirúrgica. Por lo tanto debe realizarse en todos los pacientes con fracturas, luxaciones y fracturas-luxaciones inestables, y en aquellos con déficit neurológico, preferentemente incompleto. En pacientes con lesiones estables en las radiografías simples y sin déficit neurológico, puede diferirse en función de la presión asistencial y de la disponibilidad del servicio de Radiología. En la práctica, suele indicarse

también cuando no se visualizan determinadas zonas de la columna, generalmente C1-C2 y C6-Dl. En pacientes con TCE grave deben realizarse cortes de estas zonas, si no se ven claramente en las radiografías o en el "scout" cervical, e incluso de forma rutinaria, dada la frecuente asociación de estas lesiones. La TC proporciona una excelente visualización de las estructuras raquídeas principalmente de los elementos posteriores y del canal medular, por lo que se pueden ver con nitidez los desplazamientos y fragmentos óseos que puedan estrecharlo o invadirlo. Asimismo, pueden verse fracturas o desplazamientos inadvertidos en las radiografías y nos permite valorar mejor estas lesiones, definiendo claramente las líneas de fractura, cuantificando exactamente los desplazamientos, y en definitiva, evaluando la estabilidad vertebral. Habitualmente se realizan cortes de 5 mm de espesor, aunque la exploración detallada de la columna cervical pueda requerir cortes más delgados, lo que alargará el tiempo de estudio, factor importante en el manejo de los traumatismos. Los equipos de últimas generaciones han conseguido acortar este tiempo y permiten además, la reconstrucción sagital o coronal a partir de los cortes axiales, mejorando la definición y la evaluación del canal medular. La TC es útil también para valorar las articulaciones interapofisarias y los agujeros de conjunción, así como los hematomas paravertebrales y retroperitoneales. Una ventaja adicional es la de ofrecernos información suplementaria sobre ciertas partes blandas del cuello y de las cavidades torácica y abdominal. Las fracturas horizontales que no coincidan con el plano de la TC pueden no visualizarse, como la de odontoides o algunas por compresión. Los hematomas epidurales, hematomielia y hernias discales pueden verse también con TC, aunque la RM define mejor estas lesiones. La contusión y el edema medular, las lesiones y avulsiones radiculares y los desgarros

durales requieren estudio mielográfico adicional y/o RM. La mielo-TC se realiza con inyección de contraste intratecal por punción lumbar o cervical, que obliga a la movilización del paciente o retirada del collarín cervical, además de otros inconvenientes como tiempo de estudio y reacciones adversas, por lo que generalmente no son útiles en el manejo urgente del paciente con TRM.

Ballinger, Phillip W. Cerril. Atlas de posiciones radiográficas y procedimientos radiológicos. 7.ª ed.; España: Masson. – Goaz P. W. Radiología oral (principios e interpretación). 3.ª Ed. España; ed.; Mosby.1995

11.- Cuando los resultados en la manometría esofágica muestran un aumento de la presión basal del esfínter esofágico inferior (EEI) junto a una disminución o ausencia de su relajación con la deglución, estamos frente a un cuadro de: a) Esclerodermia con afectación esofágica. b) Enfermedad por reflujo gastroesofágico. c) Espasmo esofágico difuso. d) Acalasia.

Diagnóstico de acalasia Aspecto radiográfico Un esofagograma puede mostrar la disminución de peristalsis, la dilatación del esófago proximal y el estrechamiento del esófago en su parte inferior. El paciente se traga una solución de bario, con fluoroscopia continua que son grabaciones de rayos X para observar el flujo del fluido a lo largo del esófago, sin que se observe el

movimiento peristáltico normal del esófago. Hay un agudo estrechamiento en el esfínter esofágico inferior y reducción del diámetro en la unión gastro-esofágica. La imagen que proyecta se denomina clásicamente «en pico de loro» o «en cola de ratón». Por encima de la reducción, el esófago a menudo se observa con una dilatación de diversos grados a medida que poco a poco se va estirando en el tiempo. Por la falta de movimientos peristálticos, se suele observar en la radiografía un margen entre aire y líquido. Manometría esofágica Debido a su sensibilidad, el diagnóstico es confirmado por medio de una manometría esofágica, que mide las presiones del esófago mediante una sonda nasoesofágica y permite comparar las presiones en situación basal y durante la deglución.4 Se inserta un tubo delgado a través de la nariz, y se le instruye al paciente a deglutir varias veces. La sonda mide las contracciones musculares en diferentes partes del esófago durante el acto de la deglución. La manometría revela la falla del EEI para relajarse con cada deglución y la falta de peristaltismo funcional del músculo liso en el esófago. Para descartar complicaciones se suele acudir a una endoscopia digestiva alta.

Esquema manométrico de acalasia demostrando contracciones aperistálticas, un aumento de la presión intraesofágica y el fallo de la relajación del esfínter esofágico inferior. Debido a la similitud en los síntomas, la acalasia se puede confundir con trastornos más comunes, tales como la enfermedad de reflujo gastroesofágico, la hernia de hiato, e incluso trastornos psicosomáticos. REFERENCIAS BIBLIOGRÁFICAS 1.

García Gutiérrez A. Acalasia de esófago. Disponible en: http://www.sld.cu/galerias/pdf/uvs/cirured/acalasia..pdf Consultado Mayo 27, 2006. 2. Fareras Rozman, et al. Acalasia esofágica.Tratado de Medicina Interna. 15 Edición. España: Ediciones Harcourt; 2003. Sección 2. Cap 20.p.354-9. 3. Pedroso Vázquez L, Vázquez Ríos B. Imagenología. La Habana, Cuba: Editorial Ciencias Médicas; 2005.p.58-60.

12.- Se trata de paciente masculino que acude al servicio de Oftalmología por iniciar con exoftalmos de inicio brusco ¿Qué patología es la más probable en éste paciente?

a) Miopía alta. b) Hemorragia orbitaria. c) Tromboflebitis del seno cavernoso. d) Pseudotumor inflamatorio.

EXOFTALMOS (Proptosis)

Protrusión de uno o ambos ojos a consecuencia de inflamación, edema, tumor o traumatismos de la órbita, trombosis del seno cavernoso o agrandamiento del globo ocular (como en el glaucoma congénito o la miopía magna unilateral).

Etiología, síntomas y signos En el hipertiroidismo, el edema y la infiltración linfoide de los tejidos orbitarios puede causar exoftalmos unilateral o bilateral. El comienzo brusco unilateral suele deberse a hemorragia o inflamación de la órbita o los senos paranasales. Una evolución de 2-3 sem. sugiere inflamación crónica o seudotumor orbitario (infiltración y proliferación celular no neoplásica); una progresión más lenta suele indicar neoplasia. La presencia de un aneurisma arteriovenoso en la arteria carótida interna y el seno cavernoso puede originar exoftalmos pulsátil con soplo orbitario. La proptosis después de traumatismos se debe probablemente a fístulas carótido-cavernosas, que pueden confirmarse auscultando el globo ocular. Los traumatismos e infecciones (especialmente faciales) pueden causar trombosis del seno cavernoso con exoftalmos unilateral y fiebre. Otras causas de exoftalmos unilateral son la miopía alta y el meningioma.

Diagnóstico Deben realizarse pruebas tiroideas ante todo exoftalmos prolongado sin causa aparente; si la función tiroidea es normal o el exoftalmos es de instauración rápida, debe hacerse un estudio orbitario mediante TC o RMN (v. cap. 8). El grado de proptosis puede medirse con un exoftalmómetro; si es progresiva, la exposición del globo puede originar sequedad, infección o ulceración de la córnea. Tratamiento Está en función de la etiología. En los aneurismas arteriovenosos puede ser necesaria la ligadura de la carótida común afectada o la embolización selectiva. El exoftalmos del hipertiroidismo puede desaparecer cuando se controla la función tiroidea, pero en ocasiones sigue aumentando lentamente y obliga a una descompresión quirúrgica de la órbita. Debe protegerse la córnea de la exposición si el exoftalmos es acusado. La queratitis por exposición es frecuente en el exoftalmos tiroideo. Los corticoides sistémicos suelen ser eficaces para controlar el edema y el seudotumor (p. ej., prednisona 1 mg/kg v.o. diariamente durante 1 sem, y luego a días alternos durante 5sem, reduciendo a

continuación hasta la dosis mínima para controlar el exoftalmos). Se deben extirpar los tumores.

13.- Éste hallazgo es muy frecuente en la enfermedad de Parkinson idiopática: a) Demencia al inicio. b) Pérdida de movimientos asociados en la marcha. c) Mioclonías. d) Blefaroespasmo.

La enfermedad de Parkinson es un trastorno degenerativo del sistema nervioso central. Fue descrita por primera vez en 1817 por James Parkinson, un médico británico que publicó un artículo sobre lo que llamó "la parálisis temblorosa."

La enfermedad de Parkinson (su sigla en inglés es PD), también llamada simplemente Parkinson, es la forma más frecuente de parkinsonismo, un grupo de trastornos del sistema motor. Es una enfermedad degenerativa que va progresando lentamente y cuyos síntomas, que son el resultado de una pérdida de las células encefálicas productoras de dopamina, suelen ser los siguientes: • •

• •

Rigidez del músculo - entumecimiento cuando el brazo, la pierna o el cuello se mueven de atrás hacia delante. Temblor en reposo - temblor (movimiento involuntario debido a contracciones de los músculos) que es más evidente en reposo.

Bradicinesia - lentitud para iniciar los movimientos. Inestabilidad postural - Una mala postura y falta de equilibrio que pueden causar caídas y problemas de la marcha y el equilibrio.

14.-Un varón de 65 años lee en el periódico que el antígeno prostático específico es una buena prueba de investigación para el cáncer y pide a su internista que se la haga. La prueba revela un aumento de antígeno prostático específico de 10.4 ng/ml. El tacto rectal revela una próstata de tamaño normal, pero en la ecografía se encuentra un área hipoecoica pequeña que mide 5x7 mm en el lóbulo derecho. ¿Cuál de las siguientes medidas es el siguiente paso apropiado?

a) Realizar gammagrama óseo. b) Repetir el análisis de antígeno prostático específico en tres meses para verificar si hay aumento ulterior. c) Practicar biopsia transrectal del área anormal encontrada en la ecografía. d) Comenzar el tratamiento con leuprolida de depósito.

Allen R. M. MMS Medicina Interna. 5ª. Edición. National Medical Series. Mc. Graw Hill. 2006. (capítulo 4 X C 4 a (3), b (1)). El antígeno prostático específico es una prueba que puede usarse para detección de cáncer de próstata. Sin embargo, los valores de éste también pueden estar un poco altos en la hipertrofia prostática benigna. Una ecografía transrectal puede identificar lesiones pequeñas no palpables en la exploración rectal. Si los pacientes tienen PSA alto y se confirmó en la ecografía un área anormal, se puede practicar biopsia por vía transrectal bajo guía ecográfica. En general, a los varones con cáncer de próstata se les estudia en busca de metástasis con gammagrama óseo; radiografía de tórax; tomografía computarizada de retroperitoneo y pelvis; o resonancia magnética de retroperitoneo y pelvis con énfasis en la próstata, acompañados por estudios de laboratorio. El carcinoma prostático metastásico puede tratarse con leuprolide, un agonista de la hormona liberadora de hormona luteinizante que suprime la producción testicular de testosterona. El tratamiento con leuprolide equivale a orquiectomía o a terapéutica con estrógenos en cáncer prostático o metastásico.

15.- Un varón de 18 años presenta cefalea persistente y fiebre, y después de cinco días, un cuadro de crisis convulsiva tónica focal. Una tomografía computadorizada de la cabeza muestra una lesión con reforzamiento anular en el lóbulo frontal derecho y un nivel hidroaéreo en el seno frontal del mismo lado. Lo más probable es que un aspirado neuroquirúrgico de la lesión muestre ¿cuál de los siguientes?

a) Células mononucleares pequeñas sugestivas de linfoma de Burkitt. b) Quistes de Toxoplasma gondii y taquizoítos (trofozoítos) c) Escherichia coli y Bacteroides fragilis. d) Estreptococo hemolítico a y mezcla de anaerobios.

Allen R. M. MMS Medicina Interna. 5ª. Edición. National Medical Series. Mc. Graw Hill. 2006. (capítulo 8 V A 3 a, b). El aspirado neuroquirúrgico de esta lesión probablemente muestre estreptococos hemolíticos a y anaerobios mezclados. Los abscesos cerebrales bien pueden presentarse en adolescentes. La enfermedad del lóbulo frontal suele coexistir con sinusitis e indica flora de la boca. En abscesos de origen ótico es más común encontrar Escherichia coli y Bacteroidesfragilis. El linfoma cerebral primario y la toxoplasmosis se observan rara vez en individuos inmunocompetentes. En personas con acidosis grave los cigomicetos pueden causar absceso cerebral, pero estos hongos no tienen fase de levadura.

16.- ¿Cuál de las siguientes hepatitis virales tiene el menor tiempo de incubación promedio? a) b) c) d)

Hepatitis A. Hepatitis B. Hepatitis C. Hepatitis D.

Los tiempos promedio de incubación para los virus de hepatitis son los siguientes: hepatitis A, 30 d, hepatitis B 60-90, hepatitis C 50 d, hepatitis D 60-90 d, hepatitis E, 40 d, y no se ha determinado para hepatitis G. Kasper DL, Braunwald E, Fauci AS, Hauser SL, Longo DL, Jameson JL. Harrison´s Principles of Internal Medicine. McGraw Hill. 16 Ed. 1829 p.

17.- Usted es un cirujano general y descubre que haciendo algunas modificaciones a una técnica quirúrgica disminuye el tiempo de uso de quirófano, pero quiere comparar si tiene los mismos resultados clínicos que cuando aplica la técnica clásica , ¿Qué tipo de estudio debe realizar?

a) b) c) d)

Casos y controles. Cohorte. Ensayo clínico controlado. Transversal.

Los ensayos clínicos controlados son estrategias diseñadas para evaluar la eficacia de un tratamiento en el ser humano mediante la comparación de la frecuencia de un determinado evento de interés clínico o desenlace en un grupo de enfermos tratados con la terapia en prueba con la de otro grupo de enfermos que reciben un tratamiento control.

Calva M.J.J. Estudios Clínicos Experimentales. Salud Pública de México vol.42, núm. 4, 2000 (349).

18.- ¿El principal agente etiológico de la bronquiolitis es? a) b) c) d)

Streptococcus pneumoniae.

Virus Sincitial respiratorio. Haemophilus influenzae tipo B. Rinovirus.

La bronquiolitis es reconocida como entidad clínica desde 1940 y representa la infección de las vías respiratorias bajas más frecuente en niños menores de 2 años de edad, el agente etiológico responsable del 80% de los casos es el Virus Sincitial Respiratorio. La distribución de esta enfermedad es cosmopolita y muestra un curso epidémico durante los meses de invierno e inicio de la primavera. El cuadro clínico se caracteriza por un síndrome de obstrucción bronquiolar y el diagnóstico se establece fundamentalmente con la clínica. En la mayoría de los casos es una enfermedad benigna, y sólo en ocasiones requiere hospitalización. A pesar de la frecuencia de esta enfermedad existen grandes controversias en cuanto al tratamiento más adecuado. A la fecha la mejor terapéutica que se le puede ofrecer al niño es el tratamiento de sostén. González-Saldaña N, Infectología Clínica Pediátrica, 7ª edición, páginas 245-260.

19.- ¿El síntoma cardinal en el abdomen agudo en pediatría es? a) b) c) d)

Fiebre. Dolor. Vómito. Vómito biliar.

El síntoma cardinal y más frecuente a toda edad lo constituye el dolor. En los lactantes y niños menores hay que indagar de la madre, signos indirectos de este dolor, que permitan orientación, como lo son el llanto, la posición que el niño adopta y los antecedentes en niños mayores ya es posible precisar ubicación, el punto de mayor intensidad, si el dolor es continuo o cólico y el tiempo transcurrido desde el comienzo. Un dato de interés lo constituye el antecedente de uso de analgésicos o antiespasmódicos, procedimiento bastante habitual en nuestro medio.

Bibliografía: Urgencias en Pediatría, Interamericana.McGraw – Hill. Capítulo: Urgencias Médico Quirúrgicas, Sección XXIII, pág. 716-718

20.- ¿Cual es el término que se aplica a la neoplasia benigna epitelial que forma patrones glandulares? a) Coristoma. b) Papiloma. c) Adenoma. d) Carcinoma.

Tumores benignos. Las neoplasias benignas se clasifican en neoplasias epiteliales y mesenquimatosas. Neoplasias epiteliales benignas u órganos parenquimatosos: Reciben el nombre de adenomas cuando forman patrones glandulares, así como los tumores derivados de las glándulas aunque no reproduzcan necesariamente su patrón. Se pueden observar en epitelio glandular gástrico, intestino y órganos parenquimatosos como riñón, hígado, glándulas endocrinas: • Adenomas gástricos, renal etc. Las que dan lugar a grandes masas quísticas como en el ovario reciben el nombre de cistadenoma de ovario. Algunas forman papilar que penetran en espacios quísticos se les denomina cistadenoma papilar de ovario.

• Papiloma escamoso. Neoplasias benignas originadas en epitelio escamoso de piel, cuello de útero. esófago. Crecen reproduciendo estructura en forma de dedo de guante o verrugosa que protruyen desde las superficies epiteliales. • Papiloma transicional. Estas lesiones se localizan en el epitelio transicional, vejiga.

• Pólipo. Cuando una neoplasia benigna o maligna, produce una proyección visible a simple vista sobre una superficie mucosa y crece hacia la luz, por ejemplo, en el estómago y en el colon. Anatomía patológica. AUTOR/ES: Pardo Mindán, F.J. ISBN: 9788481741735 PÁGINAS: 1364

21.- Se trata de femenino de 48 años obesa, multigesta, con malos hábitos alimenticios. Inició cuadro el día de ayer con dolor tipo cólico, en hipocondrio derecho, irradiante hacia la espalda que se presenta posterior a la ingesta de alimentos grasos, el cual se acompaña de nausea, anorexia y temperatura de 38.3 oC, niega ictericia, coluria y acolia. A la EF presenta signo de Murphy positivo a la maniobra de Pron, palpación profunda dolorosa, rebote positivo resto de exploración sin alteraciones. El diagnóstico más probable de esta paciente podría ser: a) Adenocarcinoma vesicular. b) Colecistitis aguda. c) Cáncer de la cabeza del páncreas. d) Colangitis.

La colecistitis aguda se manifiesta por dolor constante, de presentación aguda, localizado en epigastrio o hipocondrio derecho, ocasionalmente irradiado hacia la espalda, de más de 24 horas de duración, acompañado de náuseas, vómitos y fiebre (80%) de 37,5°C a 38,5°C. La mayoría de los pacientes tiene historia de cólicos hepáticos previos. La palpación abdominal descubre sensibilidad en el hipocondrio derecho, positividad del signo de Murphy y defensa muscular en esa zona (50%). En algunos casos (20%) se palpa la vesícula, lo cual es patognomónico. Este cuadro puede ser difícil de reconocer en pacientes en situación crítica. Aún en las colecistitis agudas no complicadas es frecuente el hallazgo de una ligera leucocitosis y que las tasas séricas de transaminasas, fosfatasa alcalina, bilirrubina y amilasa estén 2 a 3 veces por encima de la normalidad. Todo esto puede ocurrir en ausencia de coledocolitiasis o de pancreatitis. Ascensos superiores a los indicados deben sugerir la existencia de obstrucción biliar, colangitis ascendente o, eventualmente, de pancreatitis. El cuadro clínico de las colecistitis agudas complicadas puede ser idéntico al que originan las no complicadas, por lo que el diagnóstico frecuentemente se hace durante la cirugía o se sospecha por cambios descubiertos con alguna técnica de imagen. En algunos casos existen signos que indican la gravedad del proceso y la existencia de complicaciones supurativas (vesícula palpable, fiebre mayor de 39°C, escalofríos, adinamia y leucocitosis superior a 14.000/mm3, inestabilidad hemodinámica). La perforación con peritonitis generalizada se sospecha ante la presencia de signos de irritación peritoneal difusa, taquicardia, taquipnea, acidosis metabólica, hipotensión, shock, distensión abdominal o el hallazgo de líquido perivesi-

Clark AD, Landercasper J, Gundersen LH, et al. Effective use of percutaneous cholecystostomy in high-risk surgical patients. Arch Surg 1999;134:727-732. Lee DW, Chung SC. Biliary infection. Baillieres Clin Gastroenterol 1997;11:707-724. Lee KT, Wong SR, Cheng JS, et al. Ultrasound-guided percutaneous cholecystostomy as an initial treatment for acute cholecystitis in elderly patients. Dig Surg 1998;15:328-332. Litaker D. Preoperative screening. Med Clin North Amer 1999; 83: 1565-1581. Madrazo EG. Antibiotic dosing in renal failure. Medical Clin North Am. 1995;79:919-931.

22.- La otitis media que se acompaña con secreción y signos locales, sistémicos, o ambos, debe ser tratada con:

a) Antihistamínicos de forma exclusiva. b) Solo medidas generales. c) Drenaje, AINES y esteroides locales. d) Antibióticos, antihistamínicos y AINES.

Instituto Nacional de la Nutrición. “Salvador Zubiran” Manual de terapéutica médica y procedimientos de urgencias. Cuarta edición. Pág. 443 – 450. Mac Graw-Hill Interamericana. México. La otitis media se define como la inflamación del oído medio que se acompaña de secreción y signos locales, sistémicos, o ambos, de enfermedad aguda. Esta enfermedad predomina en la infancia, sin embargo también existe en el adulto. Su distribución tiene una clara periodicidad estacional, sobre todo en otoño e invierno. Los agentes de mayor prevalencia en nuestro país son: Streptococcus pneumoniae, Haemophilus influenzae, Morexella catarrhalis estreptococo del grupo A y S. aureuse. Los antibióticos más aceptados en estos casos son la amoxicilina, la amoxicilina con clavulanato, la calritromicina y el trimetropim con slfametoxazol. Se acepta el uso de antihistamínicos y antiinflamatorios no esteroideos para disminuir la congestión de la mucosa para resolver la obstrucción de la trompa faríngotimpánica; en realidad no se ha demostrado efecto sobre la duración de los síntomas. El drenaje del derrame solo está indicado cuando éste persiste por más de tres semanas.

23.- ¿Cuál es la característica de las heridas consideradas en la clasificación quirúrgica de heridas como limpias-contaminadas?

a) Sucede cuando previo a la herida existe evidencia de infección. b) Es la herida que se genera dentro de un quirófano. c) Es una herida limpia que se realiza en quirófano pero que involucra tejidos como gastrointestinal, respiratorio y/o genitourinario. d) Es una herida limpia realizada en quirófano que posteriormente se contamina.

-Bruce E. Jarrell, R. Anthony Carabasi, Nacional Medical Series for Independent Study. Wiliams & Wilkins, 3rd Edition: 1-25. La clasificación actual de las heridas es: Limpias, Limpias-contaminadas, contaminadas e infectadas. Las heridas limpias son aquellas que se realizan bajo técnica estéril dentro de quirófano. Las heridas limpias-contaminadas son aquellas heridas limpias que involucran tejidos potencialmente contaminantes como tubo digestivo, genitourinario y/o pulmonar. Las contaminadas son aquellas donde hay evidencia de contaminación mayor por presentarse durante una infección severa del órgano involucrado (anastomosis intestinal en peritonitis severas). La herida infectada es aquella que se presenta la infección previa a la creación de la herida como en abscesos apendiculares.

24.- Se trata de paciente femenino de 50 años de edad que acude a consulta refiriendo que desde hace varios meses presenta dificultad para tragar tanto líquidos como sólidos, así como regurgitaciones de comida sin digerir. Se determina realizar una manometría esofágica la reporta los siguientes hallazgos: ausencia de peristaltismo en el cuerpo esofágico, hipertonía y una relajación incompleta del esfínter esofágico inferior tras la deglución, con los resultados anteriores el diagnóstico más probable es:

a) Peristalsis esofágica sintomática. b) Esofagitis por reflujo. c) Acalasia. d) Esclerodermia.

La acalasia consiste en la incapacidad para relajar las fibras de músculo liso del aparato gastrointestinal en cualquier sitio de unión de una parte con otra. La acalasia esofágica, o la incapacidad del esfínter gastroesofágico para relajarse al deglutir, por degeneración de las células ganglionares en la pared del órgano.1 El esófago torácico también pierde la actividad peristáltica normal y se vuelve dilatada produciendo un megaesófago. La acalasia esofágica o simplemente acalasia es una rara enfermedad en la cual el esófago se encuentra inhabilitado para llevar el alimento hacia el estómago.

La enfermedad afecta ambos sexos y puede aparecer a cualquier edad, sin embargo se diagnostica generalmente entre la tercera y la cuarta década de la vida. Los síntomas más notables son: 1. Dolor retroesternal, que en fases iniciales es intermitente y que se va haciendo progresivo. 2. Disfagia esofágica (el alimento, una vez tragado, se "atasca" por el aumento de presión de la parte distal del esófago y el cardias). 3. En fases avanzadas, se puede dar regurgitación, dolor torácico y pérdida de peso que puede confundir con un cáncer de esófago. Disfagia que se inicia de forma brusca, generalmente a los líquidos, casi siempre relacionada con grandes emociones, de evolución caprichosa, con períodos de deglución normal y que se acompaña en ocasiones de dolor retroesternal por espasmos, y regurgitaciones de líquido claro e insípido, la cual puede ser en la noche y provocar síntomas respiratorios. La presencia de sialorrea llama la atención, así como la halitosis, casi siempre presente. En la acalasia tipo I el dolor es raro y el paciente regurgita por rebosamiento, casi siempre cuando está dormido. En la acalasia tipo II predomina el dolor al deglutir o de forma espontánea y la regurgitación es inmediata.

Gisbert J.P., Losa C., Barreiro A., Pajares J.M. Servicio de Aparato Digestivo, Hospital de la Princesa, Universidad Autonoma de Madrid. Rev Clin Esp 2000 Aug; 200(8):424-31

25.- El signo clínico de la charretera nos orienta a pensar en una luxación glenohumeral:

a) Anterior. b) Posterior. c) Superior. d) Axilar. Luxación glenohumeral Anterior: Es la forma más frecuente de luxación de hombro y una de las más frecuentes del organismo. El paciente acude con el brazo en discreta separación y rotación externa fijas: cualquier intento de movilización del hombro resulta doloroso. A la inspección se aprecia la llamada “deformidad en charretera”, por el afilamiento acromial que presenta el hombro al desaparecer el perfil redondeado de la cabeza humeral Manual CTO 7ª edición, Traumatología y Ortopedia, pág. 20.

26.- Una epistaxis anterior tiene su origen en: a) b) c) d)

En la cabeza de los cornetes. En el plexo de Keeselbach. En el cuerpo del cornete medio. En la mucosa del septum óseo y cola de los cornetes.

Epistaxis de acuerdo a la ubicación anatómica. Anterior Plexo Kiesselbach Frecuentes 90% Leve a moderada cuantía Frecuente en niños y jóvenes Fáciles de solucionar

posterior Arteria esfenopalatina Poco frecuente 10% Moderada a gran cuantía Frecuente en adultos Difícil solucionar

Boettiger O. Otorrinolaringología. II Parte: Nariz, cavidades perinasales, boca, faringe, laringe. Santiago de Chile, Saval, 2000 pp 45-56. 2. Paparella M, Shumrick D. Otorrinolaringología. Tomo III. Cabeza y cuello. Buenos Aires, Editorial Médica Panamericana, 1982 pp 1979-1993.

2. Schaitkin B, Strauss M, Houck JR: Epistaxis: medical versus surgical therapy: a comparison of efficacy, complications, and economic considerations. Laryngoscope 1987; 97: 1392-6.

4. Monux A, Tomas M, Kaiser C, Gavilan J: Conservative management of epistaxis. J Laryngol Otol 1990; 104: 868-70. 5. Waldron J, Stafford N: Ligation of the external carotid artery for severe epistaxis. J Otolaryngol

27.- La complicación más frecuente de la luxación posterior de cadera es:

a) Lesión del nervio ciático. b) Necrosis total o parcial de la Cabeza Femoral. c) Fractura central de acetábulo. d) Luxación recurrente.

Necrosis parcial o total de la cabeza femoral: La complicación más frecuente y más grave. Ocurre como consecuencia de una lesión de los vasos nutricios de la cabeza femoral, que llegan a ella a través de la cápsula articular. Junto con el desgarro ocurrido en el momento de la luxación, hay desgarro de los vasos nutricios; si el daño vascular es suficientemente importante, se corre el riesgo inevitable de una necrosis parcial o total de la cabeza femoral. Está comprobado que mientras más demore la reducción de la luxación, son mayores las posibilidades de una lesión vascular irreparable; es una de las razones que hace obligatoria la reducción precoz de la luxación. La magnitud del daño de partes blandas peri-articulares (cápsula, ligamentos, etc.), la falta de una adecuada inmovilización prolongada post-reducción, intervenciones quirúrgicas para practicar osteosíntesis de fractura cotiloídea, son otros tantos factores de riesgo de necrosis de la cabeza femoral. Sin embargo, la complicación puede presentarse, sin que haya habido ninguno de los factores agravantes mencionados. Síntomas y diagnóstico de la necrosis avascular de la cabeza femoral La precocidad con que se manifiestan los síntomas y signos, depende de la magnitud del daño vascular y de la extensión de la necrosis; y estos signos pueden aparecer meses y aun años después de la reducción de la luxación. Ello aconseja ser muy cauteloso al informar el alta de estos enfermos (accidente del trabajo, por ejemplo); en estas circunstancias es prudente dejar expresa constancia en el informe de alta, de la posibilidad que esta gravísima complicación pueda manifestarse clínica y radiográficamente hasta 2 a 3 años después del accidente.

Síntomas Dolor articular progresivo, claudicación, limitación de los movimientos, especialmente los de rotación interna y externa y de abducción de la cadera.

Radiográficamente • • •

• •

Disminución de la amplitud del espacio articular. Borramiento de la nitidez del contorno de la cabeza femoral. Alteraciones progresivas de la trama ósea de la cabeza femoral: Zonas hipercalcificadas, áreas más densas, zonas osteolíticas, aspectos microquísticos, osteolisis progresiva del cuadrante superior. Aplanamiento, desmoronamiento y finalmente destrucción total de la cabeza femoral. Cintigrafía ósea, tomografía axial computada.

La situación es progresiva e irreversible. El enfermo debe ser informado de inmediato de su situación y enviado al especialista. Generalmente el cuadro se resuelve quirúrgicamente: operación de Camera, osteotomías de centraje, artrodesis o artroplastías.

1. Bello GA, Rojas SI. Luxación traumática bilateral de cadera. Acta Ortop Mex. 2004; 18 (4): 168-172. 2. Haces F, Legorreta JG, Osuna G. Luxación traumática de cadera. Reporte de 10 casos. SMOP Revista Mexicana de Ortopedia Pediátrica [serie en Internet].1997 [citado: de 20 de diciembre 2006]; 1(1): [aprox. 7 p.]. Disponible en: http://www:smop.org.mx/nmopvlml.html 3. Álvarez Cambras R. Tratado de Cirugía Ortopédica y Traumatología. La Habana:Editorial Pueblo y Educación; 1986.p.110 4. Gustilo RB, Kyle RF,Templeman DC, eds. Fracturas y luxaciones. Madrid: Mosby Doyma Libros; 1995 5. Tachadjian MO,ed. Ortopedia pediátrica. México, DF: Interamericana Mc GrawHill;1994. 6. Salter RB,ed. Trastornos y lesiones del sistema musculoesquelético. Barcelona: Masson, S.A.; 1986.

28.- Paciente masculino de 45 años de edad post operado de interposición de injerto femoro- femoral por lesión por arma punzo cortante en el muslo derecho, presenta edema importante desde la rodilla hacia abajo, parestesias y llenado capilar retardado pero temperatura normal. El diagnostico más probable en el que hay que pensar es: a) b) c) d)

Trombosis venosa profunda. Trombosis del injerto. Síndrome compartimental. Infección de la extremidad.

FISIOPATOLOGÍA Independientemente de la causa, debido a la discordancia entre el continente y el contenido del compartimento que llevan a elevar la presión del mismo, se provoca un descenso del retorno venoso y no de la circulación arterial (ya que la tensión sistólica supera a la que existe dentro del compartimento), con lo cual aumenta todavía más la presión y conduce a una isquemia de los tejidos. CLÍNICA El principal síntoma es el dolor desproporcionado a la lesión inicial y que no responde a la elevación de la extremidad ni a analgésicos. Las localizaciones más frecuentes son la región anterior de la pierna y la cara volar del antebrazo. Otros aspectos a tener en cuenta son: -Al provocar la movilización pasiva de los músculos que forman parte del compartimento afectado, el dolor aumenta claramente de intensidad (por ejemplo, extender los dedos en el caso de la cara volar del antebrazo o extender el primer dedo del pie, si está afectada la zona anterior de la pierna). -El aspecto externo de la región es tumefacto, tenso a la palpación. La piel a veces aparece brillante. -Parestesias o hipoestesia, distales al compartimento afectado -Paresia, debida a una afectación del nervio y/o la isquemia -Pulsos periféricos presentes y relleno distal conservado. -Analíticamente puede aparecer leucocitosis, aunque es un parámentro inespecífico En casos de SC evolucionado, que es lo que debemos evitar, la clínica sería la siguiente: -Dolor muy intenso -Palidez -Ausencia de pulsos -Parestesias

-Parálisis Sigue la regla de las 5 “P”: Pain, Pallor, Pulseles, Parestesias y Parálisis En aquellos casos en los que se realizan mediciones de presión intracompartimental, se alcanzan valores >= a 30 mm de Hg.

BIBLIOGRAFÍA 1. Gómez – Castresana Bachiller F.Manual SECOT. 2004;37;412-422. 2. Sd.Compartimentales. Rockwood. 1995;1;331-340. 3. Sd. Compartimental. Médico – quirúrgica. 1994;6;825-829.

29.- Masculino de 18 años que es enviado al servicio de otorrino por presentar una tumoración en la línea media del cuello, la cual es renitente, no dolorosa y asciende al protruir la lengua. El diagnóstico clínico más probable es: a) b) c) d)

Quiste Branquial. Quiste demoide. Qiste tirogloso. Linfangioma cervical.

Los quistes del conducto tirogloso constituyen las lesiones cervicales congénitas más frecuentes en los niños. Habitualmente son detectados entre los 2 y 10 años de edad aunque un porcentaje importante no son identificados sino hasta después de los 20 años. No existe una predilección por sexo. La forma de presentación clínica clásica es la de una lesión redondeada, lisa, blanda, indolora, ubicada en la línea media y en relación al hueso hioides. Existe un pequeño porcentaje de presentaciones sublinguales o supraesternales. Como el quiste tiene fijaciones al hueso hioides y al foramen ciego de la lengua, puede ser traccionado hacia arriba, cuando el niño traga o saca la lengua. El diagnóstico es confirmado por ultrasonido, que corrobora la lesión quística. Este estudio habitualmente permite también evaluar la presencia de la glándula tiroides. Al no encontrarse ésta en el examen, es preciso realizar una cintigrafía, ya que en esos casos el único tejido tiroideo existente pudiera estar en relación al conducto o quiste tirogloso. Embriológicamente estas lesiones son consecuencia de la falla de obliteración del conducto tirogloso posterior al descenso de la glándula tiroides alrededor de la sexta semana de vida fetal. Anatomo-patológicamente los quistes se encuentran revestidos de epitelio pseudeoestratificado columnar o estratificado escamoso y glándulas secretoras de mucus.

La complicación habitual de estas malformaciones es la infección con la flora bacteriana de la boca, consecuencia de su comunicación persistente con la base de la lengua a través del foramen ciego. Los quistes infectados presentan signos inflamatorios y pueden drenar al exterior. El tratamiento recomendable es la extirpación quirúrgica y así prevenir su infección. La técnica fue descrita en 1920 por Sistrunk e incluye la extracción del centro del hueso hioides, por cuyo interior va el conducto tirogloso, y así, evitar la recidiva. La extirpación debe llegar hasta el foramen ciego lingual.

En presencia de infección quística, es necesario tratar ésta con antibióticos y posteriormente operar.

30.- Niño de 8 años de edad que presenta cuadro de ojo rojo de repetición en ojo izquierdo, y l visión disminuida a 20/80. El último cuadro tuvo 3 días de evolución. A la exploración se observa un ojo rojo moderado, apariencia despulida de la córnea y leve opacidad en área central. Tiene escasa secreción, lagrimeo y fotofobia acentuada. Al tocarla con un filamento de algodón, la córnea reacciona haciendo que se cierre el ojo más débilmente que le ojo contralateral. El diagnóstico más probable es:

a) b) c) d)

Herpes zoster. Herpes simple. Úlcera corneal. Queratocono.

Infección por herpes simple La manifestación inicial de la infección corneal por herpes simple (queratoconjuntivitis por herpes simple, queratitis) puede parecer una infección bacteriana leve porque los ojos están ligeramente doloridos, llorosos, rojos y sensibles a la luz. La inflamación de la córnea nubla la visión. Sin embargo, la infección por herpes no responde a los antibióticos, como haría una infección bacteriana, y suele empeorar cada vez más. Con mayor frecuencia, la infección produce sólo ligeros cambios en la córnea y desaparece sin tratamiento. En muy raras ocasiones, el virus penetra profundamente en la córnea, destruyendo su superficie. La infección puede ser recurrente, dañando aún más la superficie de la córnea. Las recurrencias pueden acabar en ulceración, cicatrización permanente y pérdida de la sensibilidad cuando se toca el ojo. El virus del herpes simple también puede provocar un incremento en el crecimiento de los vasos sanguíneos, empeoramiento de la visión o pérdida total de la misma.

BIBLIOGRAFÍA 1. Maertzdorf J, Van der Lelij A, Baarsma GS, Osterhaus AD, Verjans GM. Herpes simplex virus type 1 (HSV-1)-induced retinitis following simplex encephalitis: indications for brain-to-eye transmission of HSV-1. Ann Neurol 2001; 49: 104-106. 2. Ahmadieh H, Sajjadi SH, Azarmina M, Kalani H. Association of herpetic encephalitis with acute retinal necrosis syndrome. Ann Ophthalmol 1991; 23: 215-219.

3. Kamel OR, Galloway GD, Trew DR. Delayed onset acute retinal necrosis 20 years following herpetic encephalitis. Eye 2000; 14: 788-789. 4. Pavesio CE, Conrad DK, McCluskey PJ, et al. Delayed acute retinal necrosis after herpetic encephalitis. Br J Ophthalmol 1997;81:415-416.

31. - Es un factor precipitante para la encefalopatía hepática: a) Hemorragia por várices esofágicas. b) Terapia con lactulosa excesiv.a c) Acidosis metabólica. d) Ingesta reciente de alcohol.

Braunwald E, fauci A, Kasper D, Hauser S, Longo D, jameson L. Harrison Principios de Medicina Interna. 15° edición. Mc Graw Hill 2001. 2062. Uno de los factores predisponentes más frecuentes de encefalopatía hepática es la hemorragia gastrointestinal, que produce un incremento de la producción de amonio y de otras sustancias nitrogenadas. Otro factor es el aumento de proteínas en la dieta por aumento de productos nitrogenados por las bacterias del colon. Los trastornos electrolíticos, sobre todo la alcalosis hipokalémica desencadenada por un excesivo empleo de diuréticos, por paracentesis masivas o por vómitos. La alcalosis origina un aumento de la proporción de amonio no ionizado, el cual atraviesa la barrera hematoencefálica, acumulándose en el SNC. Otras causas son el empleo de opiáceos, tranquilizantes, sedantes y diuréticos, la presencia de infección y el estreñimiento, entre otras.

32.- El síndrome de Leriche se refiere a:

a) b) c) d)

Ausencia de pulsos femorales, disfunción eréctil y claudicación glútea Afectación de la aorta abdominal y arterias iliacas por enfermedad ateromatosa Imágenes radiológicas de estenosis aortoilica Afectación ateromatosa de las iliacas internas

El síndrome obliteración aortoiliaca , descrito por Graham en 1914 que debe su nombre René LeRiche (1879-1955) que operó a un joven con esta condición en 1940, pertenece al grupo de arteriopatías de las extremidades inferiores. Es un proceso crónico asociado a claudicación intermitente en las nalgas y muslos e impotencia para la erección. Los pulsos femorales y distales están ausentes. Esta condición suele afectar a sujetos varones entre 35 y 60 años. Los hallazgos patológicos característicos son lesiones arterioscleróticas progresivas de la pared de la aorta abdominal con una trombosis superpuesta que afecta en mayor o menor grado la bifucarción aortoilíaca. La obliteración se extiende en sentido proximal hasta llegar a las arterias renales a las que suele respetar. Aunque la mesentérica inferior puede estar afectada, el colon transverso no es afectado por ser suministrado por la mesentérica superior El síndrome de Leriche se manifiesta por un intenso dolor de las extremidades durante la marcha que obliga al paciente a detenerse. Esto se debido a que el aporte de sangre a las extremidades no es suficiente para hacer frente a la demanda energética que supone el aumento del trabajo muscular. El dolor desaparece cuando el sujeto se para. Por la misma razón, la cicatrización de heridas es más lenta. Como todas las arteriopatías obtructivas de las extremidades, la gravedad el síndrome de Leriche sigue la clasificación de Fontaine (*), aunque algunos autores prefieren la de Rutherford (*), más descriptiva y práctica. Diagnóstico El diagnóstico se lleva a cabo a partir de la exploración clínica. Los pacientes con síndrome de Leriche suelen presentar una desproporción del desarrollo de los miembros inferiores (siendo normales los superiores), frialdad y palidez en las piernas y en los pies. Los pulsos femorales y periféricos están ausentes. La arteriografía (*) es el método óptimo para el diagnóstico del síndrome de Leriche, en particular cuando se asocia a un TC múltiple en espiral (*). Esta técnica permite distinguir la oclusión aórtica de otras lesiones oclusivas en las arterias renales y viscerales, así como la presencia y la extensión de la circulación colateral.

Kasper DL et al. Principles of Internal Medicine. 16 th edition. McGraww-Hill. 2005

33.- La primera lesión que se observa en la retinopatía diabética temprana es: a) b) c) d)

Exudados blandos. Exudados duros. Hemorragias intrarretinianas. Microaneurismas.

La retinopatía diabética tiene cuatro etapas: 1. Retinopatía no proliferativa ligera. Esta es la etapa más temprana de la enfermedad en la que aparecen los microaneurismas. Estas son pequeñas áreas de inflamación, que parecen ampollas, en los pequeños vasos sanguíneos de la retina. 2. Retinopatía no proliferativa moderada. Según avanza la enfermedad, algunos vasos sanguíneos que alimentan la retina se obstruyen. 3. Retinopatía no proliferativa severa. En esta etapa muchos más vasos sanguíneos se bloquean, haciendo que varias partes de la retina dejen de recibir sangre. Entonces estas áreas de la retina envían señales al cuerpo para que haga crecer nuevos vasos sanguíneos. 4. Retinopatía proliferativa. En esta etapa avanzada, las señales enviadas por la retina para alimentarse causan el crecimiento de nuevos vasos sanguíneos. Esto se llama la retinopatía proliferativa. Estos nuevos vasos sanguíneos son anormales y frágiles. Crecen a lo largo de la retina y de la superficie del gel vítreo, el gel incoloro que llena el interior del ojo.

Retinopatía diabética. Forma leve de retinopatía diabética, en la que destacan micro aneurismas /micro hemorragias de predominio en arcada temporal superior, junto con exudados duros que se acercan a la mácula. 1. The Diabetes Control and Compications Trial Research Grop. The Effect of Intensive treatment of Diabetes on the development and progression of long term complications in insulin dependent Diabetes Mellitus N Engl J. Med; 1993 ; 329 : 977-986. 2. Early treatment Diabetic Retinopathy Study Research Group Early photocoagulation for diabetic Retinopathy ETDRS Report 9. Ophthalmology, 1991 ; 98 : 1316-26. 3. Kahn HA Hiller R. Blindness caused by diabetic retinopathy Am. J. Ophthalmol, 1974; 78 ; 58-67. 4. Ladas ID. Theossiadis GA Long term efectiveness of modified grid photocoagulation for diffuse Macular edema Acta Ophthalmol, 1993 ; 71(3) ; 393-7.

5. Klein R, Klein BEK , Moss SE , et al The Eisconsin Epidemiology Group Ophthalmology, 1984 ; 91 : 1464-74

34.- ¿Cuál es la base de la terapéutica específica de la pancreatitis aguda? a) Antibióticos profilácticos. b) Analgésicos. c) Ayuno. d) Aspiración nasogástrica.

Tratamiento en pancreatitis aguda. El 80 % de los pacientes con pancreatitis aguda se tratan mediante medidas de sostén, es imprescindible la suspensión de la vía oral, por lo tanto hidratación intravenosa, analgésicos y alimentación parenteral cuando la vía oral se restringe más allá de la semana.

La indicación de la suspensión de la vía oral se basa en el dolor y la intolerancia digestiva. No debe ser prolongada y se debe restablecer secuencialmente luego de 48 horas sin dolor. La alimentación parenteral no tiene ningún sentido si el restablecimiento de la via oral se realiza dentro de los primeros 7 días. De no ser así se sabe que esta patología grave provoca una agresión severa que determina un estado híper catabólico por lo tanto debe implementarse soporte nutricional para evitar la desnutrición y las complicaciones que ella trae consigo (alteración de la modulación de la respuesta inflamatoria, translocación bacteriana, inmuno supresión, etc.) •

Feldman: Sleisenger and Fordtrans Gastrointestinal and liver diseases, 8TH ed 2006 Saunders, an imprint of Elsevier

35.- En el servicio de traumatología se valora a un paciente masculino de 23 años que presenta una tumoración en hueco poplíteo de varios años de evolución, lesión indolora y que cambia de tamaño, su diagnóstico clínico es:

a) Quiste de Baker. b) Hérnia muscular de gemelo externo. c) Sarcoma de partes blandas. d) Osteocondroma femoral.

El quiste de Baker o quiste poplíteo, es una masa de tejido blando localizada en la cara posterior de la rodilla, la cual se llena con líquido gelatinoso proveniente de la articulación de la rodilla. Se ubica con mayor frecuencia del lado interno y posterior de la misma. Se producen por una debilidad de la membrana sinovial que recubre la articulación de la rodilla, generando una “hernia” de la articulación. Desde el punto de vista histológico se clasifican en: 1. Fibrosos: tienen una pared limitante definida y una superficie interna lisa brillante. 2. Sinoviales: tienen una pared gruesa (2 a 5 mm), es menos brillante. 3. Inflamatorios o Transicionales: Tienen una pared muy gruesa (mas de 5 mm) y su consistencia es blanda. En los niños, la molestia inicial es una masa en la región posterior de la rodilla (hueco poplíteo). Generalmente en niños pequeños, no se observan otros síntomas como rigidez o dolor local, aunque en adolescentes si se presentan. La masa aumenta de tamaño cuando la rodilla esta en hiperextensión y desaparece cuando la rodilla esta en flexión. La masa es

dura, firme y se puede evidenciar con la transiluminación (colocar una luz fuerte sobre la masa, evidenciándose la presencia de material líquido). Es de suma importancia revisar con gran detenimiento la articulación de la rodilla, con la finalidad de buscar signos sugestivos de sub luxación o artrosis. El diagnostico se realiza cuando se examina al niño, la madre refiere que en ciertas ocasiones, sobre todo después de actividad física esta de mayor tamaño, al levantarse es pequeña y aumenta su tamaño durante el día. Se debe realizar un Rx de la rodilla, donde se evidenciara el aumento de los tejidos blandos y la presencia de una masa de densidad liquida. El ecosonograma ayuda a delimitar los bordes del mismo. La Tomografía axial computarizada y la Resonancia Magnética Nuclear señalaran con claridad si el quiste tiene densidad liquida, define la relación con el resto de las estructuras y orienta sobre el grosor de la pared del quiste. El tratamiento inicial es Observación, ya que el quiste generalmente tiende a desaparecer, hay que explicarle a los padres la naturaleza de la lesión y que no existe posibilidad de malignización, en trabajos de investigación se ha determinado que aproximadamente el 80% de los quistes desaparecen en el transcurso de 2 a 3 años, en caso de que la masa crezca de manera importante, que limite la actividad del niño o que comience a presentar dolor, la indicación quirúrgica no tiene discusión. No se recomienda aspirar el quiste o la infiltración con esteroides debido a que con estas técnicas siempre tiende a reproducirse.

Atlas IRM de Ortopedia y Traumatología de Rodilla Autor: Peter Teller Editorial: AMOLCA Nº Edición: 1ª Idioma: Castellano

Año: 2005 Nº Volúmenes: 1

36.- El riesgo de adquirir infección por virus del papiloma humano entre estudiantes universitarios se mide en un estudio epidemiológico a través de: a) La incidencia acumulada. b) La densidad de incidencia.

c) La tasa de incidencia dividida entre la prevalencia. d) Prevalencia.

La incidencia de una enfermedad se mide de dos formas: mediante la densidad de incidencia que expresa la ocurrencia de la enfermedad entre la población en relación con unidades de tiempo-persona, por lo que mide la velocidad de ocurrencia de la enfermedad y la incidencia acumulada que expresa el volumen de casos nuevos ocurridos en una población durante un periodo, y mide la probabilidad o riesgo de los miembros de una población, de contraer una enfermedad en un periodo específico.

Moreno A. Principales medidas en epidemiología. Rev Salud Pública Mex, 2000;42(4): 343

37.- Las dosis fisiológicas sustitutivas por día de la prednisona son: a) b) c) d)

5 a 7.5 mg. 10 a15 mg. 20 a 25 mg. 75 a 100 mg.

El fármaco que debe preferirse para tratar la insuficiencia suprarrenal es la hidrocortisona. En la mayoría de los adultos la dosis (según su tamaño) es de 20 a 30 mg/día. Sin embargo en México no es fácil conseguir hidrocortisona vía oral, por lo que los pacientes se tratan con prednisona. La dosis sustitutiva es de 5 a 7.5 mg/día. Williams GH, Dluhy RG. Enfermedades de la corteza suprarrenal. En Jameson JL (ed): Harrison. Endocrinología. 1a ed. Madrid. MacGraw-Hill España, 2006: 137-138.

38.- La técnica actual para la cirugía de catarata se llama: a) Extracción extracapsular. b) Vitrectomía. c) Facoaspiaración.

d) Facoemulsificación. .

. FACOEMULSIFICACION: (Cirugía de Catarata) Consiste en la extracción de la Catarata a través de una incisión pequeña de 3 mm. en la cornea, el tejido transparente y mas anterior del ojo, de modo que no existe ni el mas mínimo sangrado. Por lo tanto la cirugía puede efectuarse en la mayoría de los casos con anestesia tópica, esto es, utilizando solamente gotas en el ojo para anestesiarlo. Se introduce por la incisión antes mencionada una sonda de ultrasonido (figura1) que va fragmentando la catarata y que se extrae aspirando los fragmentos con la misma sonda. En seguida, y mediante un inyector se introduce un lente intraocular que por ser plegable es capaz de penetrar a través de esa pequeña incisión (figura 2). Esta incisión, que en la mayoría de los casos no requiere de sutura o tal vez un solo punto de sutura, permite mayor seguridad durante el acto quirúrgico y una recuperación mas rápida.

Si se utilizó anestesia tópica es factible que el paciente no use oclusión o parche y la visión será inmediata después de la cirugía (figura 3). Sin embargo puede usar la oclusión al menos por un día. Una ventaja mas es que siendo una incisión pequeña es menos dolorosa y produce menos astigmatismo corneal (irregularidad). Existe otra técnica de incisión mas pequeña de 1.5 mm. pero finalmente al introducir el lente se tiene que ampliar a los 3 mm.

1.

Coret Novoa A, Coret Moreno A. Tiempos de facoemulsificación. Barcelona: EDIKAMED, 1995; 19-24.

2. El equilibrio dinámico. Parámetros de facoemulsificación. Manual AlconCusí.

Barcelona, 1996.

3. Black H. Phaco tool's efficiency allows the surgeon to use more vacuum power. Ocul

Surg News 1996; 4: 37-41.

39.- El primer anticuerpo producido pos las amígdalas palatinas al exponerse a un antígeno es: a) b) c) d)

IgE. IgG. IgA. IgM.

La IgA está presente en el plasma y en secreciones. Es la ¡nmunoglobulina predominante en las secreciones externas donde actúa localmente neutralizando posibles patógenos. Manual CTO de Medicina y Cirugía. Séptima Edición. McGraw Hill, pgs 824.

40.- Indique en cuál de las siguientes circunstancias considera indicada la colecistectomía:

a) b) c) d)

Colelitiasis asintomática en un paciente diabético. Colesterolosis. Vesícula en porcelana. Colelitiasis asintomática en un paciente cirrótico.

COLECISTITIS CRÓNICA EN PORCELANA – Calcificación en el fondo, cuerpo y cuello. – Poco frecuente. – Se vincula con cáncer. Cáncer de Vesícula El cáncer de vesícula, se asocia fuertemente a la colelitiasis. El riesgo de presentar cáncer es directamente proporcional al tiempo que el paciente ha sido portador de una colelitiasis y al tamaño de los cálculos. La vesícula en porcelana y la vesícula escleroatrófica tienen

riesgo elevado de presentar un cáncer (hasta 20%). Probablemente la inflamación crónica de la vesícula induciría cambios que provocarían la transformación progresiva del epitelio, comenzando con lesiones premalignas (displasia, hiperplasia atípica, adenoma) hasta el cáncer avanzado. • Todo paciente con vesícula calcificada (vesícula en porcelana) es aceptable el riesgo quirúrgico con el propósito de prevenir el carcinoma de vesícula como complicación tardía. Algunas de las indicaciones de Colecistectomía -Expectativa de vida > 20 años -Cálculo >2cm o < de 3mm. -Cálculos radiopacos -Cálculos calcificados -Pólipos vesiculares -Vesícula no funcionante -Vesícula en porcelana -Diabetes -Enfermedad crónica concomitante grave -Mujer < 60 años -Zonas con alta prevalencia de cáncer de vesícula. REFERENCIAS BIBLIOGRÁFICAS 1. Feldman. Sleisenger & Fordtran's Gastrointestinal and Liver Disease, 7 ed.New York: Elsevier; 2002. 2. Townsend S.Textbook of Surgery. 16 ed.New York: W. B. Saunders Company; 2000.

3. Goldman C. Textbook of Medicine. 21 ed. New York :W. B. Saunders Company;2000. 4. Rakel C. Current Therapy. 55 ed. New York: Elsevier; 2003. 5. Cotran R. Pathologic Basis of Disease. 6 ed New York: W. B. Saunders Company;1999. 6. Duthie. Practice of Geriatrics. 3 ed.Washigthon: W. B. Saunders Company ;1998. 7. Ferri F. Clinical Advisor: Instant Diagnosis and Treatment.New York: Mosby, Inc; 2000. 8. Noble. Textbook of Primary Care Medicine. 3 ed.New York: Mosby, Inc;.2001

41.- ¿Al realizar exploración física a un paciente, usted determina que éste, comprende pero no puede articular palabra, que tipo de afasia tiene?

a) Wernike. b) Broca. c) Global. d) Anómica.

• • • • • • • •

Afasia De broca, motriz o de expresión (no fluente) De Wernicke (no comprende) De conducción (no denomina – no repite) Anómica (no denomina) Transcortical motora (no fluente) Transcortical sensitiva (no denomina – no comprende) Global (no fluente-no denomina-no repite-no comprende) Talámicas y putaminales (lenguaje espontaneo incomprensible)

1.Aronson A y cols. Examen clínico neurológico, 3ª Edición. La Prensa Médica Mexicana, México, 1995. 2. Uribe CS, Arana A, Pombo PL. Neurología, 5ª Edición. Corporación para investigaciones biológicas. Colombia, 1996. 3.-Adams R, Víctor M. Principles of Neurology. Mc Graw Hill. 7th ed. USA 2001. 4.-Bradley W.G. Neurology in clinical practice. Butterworth Heinemann. 4th ed. Philadelphia USA, 2004. 5.-Silberstein SD, Lipton RB, Goadsby PJ. Headache in clinical practice. Isis Medical

Media. Oxford University Press, UK, 1998. 6.-Barinagarrementería F. Cantú C. Enfermedad vascular cerebral. McGrawHill Interamericana. México D.F. 1997.

42.- Se trata de paciente 69 años, sin antecedentes neurológicos, psiquiátricos ni tratamientos farmacológicos previos, inicia aproximadamente hace 9 meses con deterioro mental progresivo, fluctuaciones en su nivel de atención y rendimiento cognitivo, alucinaciones visuales y, en la exploración neurológica, signos parkinsonianos leves. El diagnóstico más probable es:

a) Enfermedad de Alzheimer.

b) Demencia con cuerpos de Lewy. c) Enfermedad de Huntington. d) Demencia vascular.

DIFERENCIAS ENTRE DCL Y EA Síntomas neuropsiquiátricos Alucinaciones visuales

+++ (precoces y persistentes)

+ (tardías)

Ideas delirantes

+++

++

Depresión

++

++

Apatía

++

++

Temblor

++

-

Rigidez

+++

+

Bradicinesia

+++

+

Signos motores extrapiramidales

Fluctuaciones clínicas Fluctuación cognitiva

+++ (Prominente, grave y precoz)

+

Neuropsicología Trastorno precoz en:

Atención, visuopercepción

Atención, memoria declarativa

Neuroimagen Atrofia cerebral global

++

++

Atrofia del lóbulo temporal medio

+

+++

Hipoperfusión del lóbulo occipital

+++

-

Deterioro de la actividad

+++

-

++

+++

+

+++

Cuerpos de Lewy subcorticales

++

-

Cuerpos de Lewy corticales

+++

-

Déficit colinérgico

+++

++

Déficit dopaminérgico

++

-

++

++

dopaminérgica Neuropatología y bioquímica Densidad de placas seniles Densidad de ovillos neurofibrilares

Genética Sobrepresentación de APOE-ε4

+++ Manifestación típica ++ Habitualmente presente + Presente - Inusual

Tabla adaptada de: U.P.Mosimann, I. McKeith. Dementia with Lewy bodies: diagnosis and treatment. Swiss Medical Weekly 2003;133:131-142

De la Vega, R. y Zambrano, A.

Demencia con cuerpos de Lewy [en línea]. La Circunvalación del hipocampo, 2009 .

43.- La enfermedad de Sever es un tipo de Osteocondritis que afecta a:

a) Calcáneo. b) Tuberosidad anterior de la tibia. c) Escafoides tarsal. d) Escafoides carpal.

Las osteocondrosis son alteraciones de uno o más centros de osificación, caracterizados por degeneración secuencial o necrosis aséptica y recalcificación. Existen varios factores implicados en su génesis: alteraciones de la vascularización, factores mecánicos, núcleos accesorios de osificación, influencia genética. Enfermedad de Sever se localiza en el calcáneo, epidemiología predomina en el varón, entre los 6-7 años. Manual CTO 5ª edición, Traumatología y Ortopedia, pág 45-46.

44.- Un varón de 30 años presenta dolor y tumefacción del testículo derecho. Su médico solicita un ultrasonido que revela una masa testicular de 2 x 2.5 cm. Se realizan una exploración inguinal y una orquiectomía. El estudio histopatológico reveló un seminoma puro. Una tomografía computadorizada de tórax, abdomen y pelvis mostró dos ganglios retroperitoneales de 3 cm que están aumentados de tamaño. La biometría hemática, la química sanguínea y los marcadores tumorales están todos dentro de los límites normales. ¿Cuál de los siguientes seria la mejor conducta? a) Extirpación quirúrgica de toda la enfermedad. b) Radioterapia. c) Quimioterapia. d) Observación.

Allen R. M. MMS Medicina Interna. 5ª. Edición. National Medical Series. Mc. Graw Hill. 2006. (capítulo 4IX G 2). El cáncer testicular es el más común en varones adultos jóvenes variedades más frecuentes son seminomas y tumores de células germinales no seminomatosos; ambos sor rabies aun en etapas avanzadas. Los seminomas son muy sensibles a la radioterapia; por tanto, los pacientes enfermedad de etapa II (la limitada a testículo y a ganglios por abajo del diafragma) pueden tratarse con < bajas de radiación. Debido a la toxicidad de la médula ósea producida por la radioterapia mediastínica, y está indicada la radiación profiláctica del mediastino. En este contexto, los ganglios linfáticos retroperitoneales aumentados de tamaño denotan la existencia de enfermedad metastásica y está indicado el tratamiento.

45.- Sitio de pérdida de neuronas en la enfermedad de Alzheimer:

a) Corteza integración sensitiva. b) Corteza de asociación frontal, parietal y temporal. c) Corteza de asociación occipital y lóbulo limbito.

™

d) Corteza somato sensorial auditiva y visual. Embolismo 80%,

Toda enfermedad bien delimitada se caracteriza por unos síntomas concretos y definidos, unas lesiones específicas y una causa que origina tanto aquéllos como éstas. La enfermedad que el neuropsiquiatra alemán Alois Alzheimer describió por primera vez en 1906 es al día de hoy la causa más frecuente de demencia entre las personas mayores. El Alzheimer es una enfermedad progresiva y todavía incurable que destruye la inteligencia y produce efectos desastrosos en el enfermo, su familia y la sociedad. La enfermedad de Alzheimer (AD) es un desorden neurodegenerativo en el sistema nervioso central (SNC) que ocurre por lo general en personas de avanzada edad, caracterizado por el progresivo deterioro en las funciones mentales, que aflige en sólo los Estados Unidos a 2.5 millones de personas y se calcula que para el año 2040, 5 millones sufrirán esta enfermedad. El costo que se invierte por año en E.U. en el tratamiento a las personas que padecen AD es de 40.000 millones de dólares, por lo que se realizan numerosas investigaciones en ésta enfermedades para disminuir el costo social que ésta representa.

Las lesiones del Alzheimer El diagnóstico de certeza de enfermedad de Alzheimer requiere realizar la autopsia cerebral. Exige la comprobación microscópica de las lesiones características.

Macroscópicamente, en el cerebro hay disminución del peso y volumen, aumento del tamaño de los ventrículos y de la profundidad de los surcos. La atrofia es particularmente llamativa en la base de los lóbulos temporales, hipocampos, circunvoluciones parahipocámpicas, lóbulos parietales y frontales. La patología microscópica incluye:

Placas seniles (PS), (difusas y clásicas). • Ovillos neurofibrilares (DNF). • Hilos del neurópilo (HN). • Pérdida neuronal y de sinapsis (degeneración neuronal). • Depósitos de amiloide en el cerebro y vasos sanguíneos cerebrales y meníngeos. Degeneración gránulo-vacuolar en las células piramidales del hipocampo. Presencia de cuerpos de Hirano. Gliosis reactiva. Aumento de las células de la microglía. Alteraciones pseudo-espongiformes. •

• • • • •

Microscópicamente se comprueba atrofia y pérdida neuronal, disminución de sinapsis, placas seniles, ovillos neurofibrilares, neuritas distróficas, degeneración gránulovacuolar, cuerpos de Hirano, marcada reacción inflamatoria glial con astrocitosis reactiva y angiopatía amiloide. En el Alzheimer hay una intensa pérdida de neuronas y sinapsis y una gran atrofia cerebral. Las lesiones características que se ven al examinar el tejido cerebral con el microscopio son conocidas con el nombre de placas seniles o neuríticas esparcidas abundantemente en la corteza cerebral y situadas fuera de las neuronas y ovillos neurofibrilares que aparecen dentro de las neuronas. Se presenta pérdida de neuronas y de conexiones neuronales, pérdida de peso y aumento de contenido de agua en el cerebro, disminuye la densidad de terminales presinápticas, disminución de acetilcolina transferasas (CAT) en el núcleo basal de Meynert además se pierden otros neurotransmisores como serotonina en un 70 %, GABA en 5 %, somatostatina en 40-60 % y glutamato.

BIBLIOGRAFÍA: AINNE, J., SAHLBERG, N., RUOTTINEN, H., NAGREN, K. and LEHIKOINEN, P. 1998.

Striatal uptake of the dopamine reuptake ligand [ 11 C] b - CFT is reduced in Alzheimer’s disease assessed by positron emission tomography Neurology. 50: 152-156. AUNOZ, D. 1998. Is exposure t. is aluminum a risk factor for the developmetn of AD. Arch Neurol. 55: 737 – 739. BLACKER, D., HAINES, J., RODES, C., TERWEDOW, H., GO, R., HARRELL, F., CRAFT, S., PESKIND, G., SCHWARTZ, M., SCHELLENBERG, G., RASKIND, M. and PORTE, O. 1998. Cerebrospinal fluid and plasma insulin levels in Alzheimer’s disease. Relation ship to severity of dementia and apolipoproteina E. Genotype. Neurology. 50: 164 – 168. CRAFT, S., TERI, L., EDLAND, S., KUKULL, W., SCHELLENBERG, B., MCCORMICK, W., BOWEN, J. and LARSON, E. 1998. Accelerated declined in apolipoprotein E E4 homozygotes with Alzheimer’s disease. Neurology. 51: 149 – 153. CUMMINGS., VINTERS, H., COLE, G. and KHACHATURIAN, S. 1998. Alzheimer’s Disease Etiologies, Pathophysiology, Cognitive Reserve and Treatment Opportunities. Neurology. 51 (supply 1):52 – 517. FORBES, W. and HILL, G. 1997. Is exposure t. is aluminum a risk factor for the developmetn of AD. Arch. Neurol. 55: 740 – 741. FRISONI, G. and BIANCHETTI, A. 1998. The aded value of MRI measure of atrophy in the diagnosis of Alzheimer’s disease. Neurology . 51:1238. GROSSMAN, M., PAYER, F., ONISHI, F., D’ESPOSITO, M., MORRISON, D., SADEK, A. and ALAVI, A. 1998. Language comprehesion and regional cerebral defects in frontstemporal degeneration and Alzheimer’s disease. Neurology. 50:157 – 163. HACHINSKI, V. 1998. Aluminun exposure and risk of 40. Arch. Neurol. 55:742. LEVALLOIS, P. 1997. AD and aluminium. Neurology. 48:1142.

46. - Paciente obesa, con fractura de cadera que será intervenida quirúrgicamente para reemplazo total de cadera se considera: a) b) c) d)

Paciente de riesgo bajo para trombosis venosa profunda. Paciente de riesgo alto para trombosis venosa profunda. Paciente con riesgo de trombosis arterial. Paciente con riesgo muy bajo de TEP.

La fisiopatología de la TVP se resume mediante la tríada de Virchow: estasis sanguíneo, daño endotelial e hipercoagulabilidad. Estas 3 circunstancias aisladamente o en asociación, intervienen en el desarrollo de un trombo. Los factores de riesgo enumerados a continuación aumentan la probabilidad de desarrollar trombosis mediante uno o más de los mecanismos de la tríada (Kahn S, 1998): 1. Cirugía mayor, especialmente la cirugía ortopédica, pero también los pacientes sometidos a cirugía abdominal, neurocirugía. 2. Neoplasias malignas, el riesgo aumenta en los pacientes que reciben quimioterapia activa. 3. Infarto Agudo de Miocardio. 4. Síndrome Nefrótico. 5. Ictus isquémico, preferentemente en el miembro hemipléjico. 6. Inmovilización prolongada. A mayor tiempo de inmovilización mayor riesgo, aunque se ha documentado una incidencia de TVP del 13% en pacientes encamados durante 8 días. 7. TVP o Embolismos Pulmonares (EP) previos. El riesgo se debe probablemente a la persistencia de obstrucción al flujo y/o daño en las válvulas venosas tras la TVP anterior. 8. Embarazo y postparto. 9. Anticoncepción oral y Terapia Hormonal Sustitutiva (THS). 10. Alteraciones congénitas o adquiridas que produzcan hipercoagulabilidad: o Deficiencia congénita de Proteína C, Proteína S y Antitrombina III. o Resistencia a la Proteína C activada: Esta alteración se encuentra en el 5% de la población general y en el 20-40% de los enfermos con TVP. o Hiperhomocisteinemia o Disfibrinogenemia o Presencia de Anticuerpos Antifosfolípido Estos trastornos se asocian a TVP recurrentes, o en localizaciones atípicas o de aparición a edades tempranas (antes de los 45 años).

Bibliografía Ebell MH. Evaluation of the patient with suspected deep vein thrombosis. J Fam Pract. 2001 Feb;50(2):167-71 [PubMed] [Texto completo] Gabriel Botella F, Labiós Gómez M, Brasó Aznar JV. Trombosis venosa profunda: presente y futuro. Med Clin 2000; 114: 584-596. [PubMed] Gorman WP, Davis KR, Donnelly R. ABC of arterial and venous disease. Swollen limb-1: General assessment and deep vein thrombosis. BMJ 2000; 320: 1453-1456. [PubMed] [Texto completo]

Green L, Fay W, Harrison V, Kleaveland M, Wahl R, Wakefield T, Weg J, Williams D. Venous thromboembolism (VTE) [Internet]. Ann Arbor (MI): University of Michigan Health

System 2004 [acceso 18/3/2007] Disponible en: http://cme.med.umich.edu/iCME/vte04/ [NGC] Kahn S. The clinical diagnosis of deep venous thrombosis. Arch Intern Med 1998; 158: 2315-2323. [PubMed] Lowe G et al. Antithrombotic therapy. A national clinical guideline [Internet]. Scottish Intercollegiate Guidelines Network. March 1999, revised 2001 [acceso 18/3/2007] Disponible en: http://www.sign.ac.uk/pdf/sign36.pdf Ninia J. Thrombosis and thrombophlebitis [Internet]. The American College of Phlebology [acceso 18/3/2007] Disponible en: http://www.phlebology.org/syllabus12.htm

47.- Femenino de 28 años manifiesta que inicialmente presentó un enrojecimiento, sensibilidad y dolor en el borde externo del párpado. Actualmente cursa con orzuelo, usted decide el siguiente tratamiento por ser el de elección:

a) b) c) d)

Compresas frías. Drenaje. Compresas tibias y antibióticos tópicos. Resección amplia.

Orzuelo Un orzuelo es una infección, en general provocada por un estafilococo, de una o más de las glándulas que se encuentran en el borde del párpado o por debajo de éste. Se forma un absceso que tiende a romperse y, en consecuencia, genera una pequeña cantidad de pus. El orzuelo a veces se forma al mismo tiempo que la blefaritis o bien como resultado de ésta. Una persona puede tener uno o dos orzuelos en toda su vida, pero otras los desarrollan repetidamente. El orzuelo en general se manifiesta primero con un enrojecimiento, sensibilidad y dolor en el borde externo del párpado. Luego, una pequeña área se torna redondeada y sensible y se hincha. El ojo puede lagrimear, volverse muy sensible a la luz intensa y provocar la sensación de que hay algo en su interior. Generalmente, sólo una parte muy pequeña del párpado se hincha, pero a veces se inflama en su totalidad. En general aparece un diminuto punto amarillento en el centro de la zona hinchada. A pesar de que se recurre a los antibióticos, no parecen ser demasiado útiles en estos casos. El mejor tratamiento consiste en aplicar compresas calientes durante 10 minutos varias veces al día. El calor ayuda a que el orzuelo madure, se rompa y drene. Cuando se forma un orzuelo en una de las glándulas más profundas

del párpado, una afección llamada orzuelo interno, el dolor y los demás síntomas suelen ser más intensos. El dolor, el enrojecimiento y la hinchazón suelen aparecer sólo en un área muy pequeña, en general en el borde del párpado. Como esta clase de orzuelo rara vez se rompe por sí solo, el médico puede abrirlo para drenar el pus. Los orzuelos internos suelen ser recurrentes.

48.- Un niño de 6 años de edad refiere un desarrollo insidioso de “cojeo” con disminución en la movilidad de la cadera. Se queja ocasionalmente de dolor en la rodilla ipsilateral. Presenta una marcha anti-álgica. LA EF es normal, pero la movilidad pasiva de la cadera es limitada. El niño se encuentra afebril, y los padres refieren que su marcha y nivel de actividad eran normales hasta antes de este problema. No ha tenido fiebre recientemente. ¿Cuál es el diagnóstico más probable?

a) b) c) d)

Necrosis avascular de la epífisis capital femoral. Displasia de cadera. Osteomielitis hematógena de cabeza femoral. Artritis séptica.

La necrosis avascular de la cabeza femoral se manifiesta con dolor inguinal que se irradia de modo intermitente hacia la región anteromedial del muslo. Los Pacientes pueden presentar una marcha antiálgica, cojera por insuficiencia del glúteo menor o limitación del arco de movilidad, en especial para la flexión, la abducción y la rotación medial, con un chasquido provocado por la rotación lateral de la cadera en flexión y abducción, sobre todo al levantarse desde la posición sentada. Diagnóstico El diagnóstico precoz requiere un índice de sospecha elevado en Pacientes que refieren dolor, en particular en caderas, rodillas u hombros. Las pruebas diagnósticas dependen de la fase en que esté el trastorno. La RM es la técnica más sensible y específica y se debe emplear para el diagnóstico en fases muy tempranas, cuando puede evitarse el colapso de la cabeza femoral o de otro hueso. Si no está clara la duración de la enfermedad, se deben obtener radiografías o TC para descartar enfermedad avanzada. La gammagrafía ósea es más sensible que la radiología, aunque inespecífica, por lo que se emplea menos que la RM. Los hallazgos radiográficos pueden no ser aparentes desde meses hasta 5 años después del comienzo de los síntomas. Los signos radiográficos comienzan con una esclerosis sutil del hueso. En la cabeza femoral se sigue de una radiotransparencia característica (el signo de la luna creciente); continúa con el colapso del hueso (aplanamiento de la cabeza femoral) y

por último de estrechamiento del espacio articular y cambios artrósicos en los huesos enfrentados en la articulación.

1. López–Durán L. Necrosis óseas avasculares. En: López-Duran L. Traumatología y Ortopedia. 2ª ed. Madrid: Luzán, 1995; 5: 115-7. 2. Coombs RR, Thomas RW. Avascular necrosis of the hip. BrJ Hosp Med 1994; 51(6): 27580. 3. Mitchell MD, Kundel HL, Steimberg ME, Kressel HY,Alavi A, Axel L. Avascular necrosis of the hip: comparison of MR, Ct and scintigraphy. AJR Am J Roentgenol 1996;147 (1): 67-71. 4. Markisz JA, Knowles RJ, Altchek DW, Schneider R, WhalenJP, Cahill PT. Segmental patterns of avascular necrosi of the femoral heads: early detection with MR imaging.Radiology 1987; 162 (3): 717-20. 5. Mitchell DG, Kressel HY, Arger PH, Dalinka M, SpritzerCE, Steimberg ME. Avascular necrosis of the femoral head:morphologic assesment by MR imaging, with CT correlation.Radiology 1986; 161 (3): 739-42. 6. Manero FJ, Castellano MA, Manero I. Radiología ósea.Medicina Integral 1997; 30 (5): 217-9. 7. Dutkowsky JP. Trastornos no traumáticos diversos. En: Crenshaw AH. Campbell. Cirugía Ortopédica. 8ª ed. Buenos Aires: Panamericana, 1994; 1922-4. 8. Aaron RK, Lennox DW, Bunce GE, Ebert T. The conservative treatment of osteonecrosis of the femoral head: a comparison of core compresion and pulsing electromagnetic fields. Clin Orthop 1989; 249 (1): 209-18. 9. Rioja Toro J. Magnetoterapia. En: Rioja Toro J. Electroterapia y electrodiagnóstico. 2ª ed. Valladolid: Universidad de Valladolid 1996; 299-309. 10. Xenakis TA, Beris AE, Malizos KK, Koukoubis T, Gelalis J, Soucacos PN. Total hip arthroplasty for avascular necrosis and degenerative osteoarthritis of the hip. Clin Orthop 1997; 341: 62-8.

49.- Un hombre de 55 años se le diagnostica HPB, se niega a tomar tratamiento farmacológico y elige una RTUP, cuál es la complicación más frecuente de este procedimiento. a) b) c) d)

Contractura de cuello vesical. Impotencia. Incontinencia. Eyaculación retrógrada.

Resección Transuretral (RTU): es el gold standard de las intervenciones. __ En general es una cirujía corta.

__ 90% mejoría importante de los síntomas. __ Morbimortalidad baja del 0,2%. __ El 20 % debe reoperarse a los 10 años aproximadamente. __ Complicaciones: impotencia 10%, eyaculación retrograda 50%, incontinencia 4%.

BIBLIOGRAFIA: 1. Medina JJ, Parra RO, Moore RG. Benign prostatic hyperplasia (the aging prostate). Med Clin North Am 1999 Sep;83(5):1213-29 2. Oesterling JE. Benign prostatic hyperplasia. Medical and minimally invasive treatment options. N Engl J Med 1995 Jan 12;332(2):99-109.

50.- El siguiente fármaco es utilizado en el manejo de la hiperplasia prostática benigna cuya acción es inhibición de la enzima 5 a-Reductasa: a) Prasozina. b) Finasterida. c) Leuprolide. d) Danazol. . Finasterida La finasterida es un fármaco antiandrogénico derivado no hormonal de los esteroides. No es un antagonista androgénico propiamente dicho como la flutamida, al no bloquear los receptores androgénicos en el citoplasma ni en el núcleo celular.

Mecanismo de acción Es un inhibidor competitivo de la enzima 5-alfa-reductasa de tipo II que transforma la testosterona en dihidrotestosterona, su molécula activa. Por lo tanto la finasterida produce una profunda disminución de la conversión de testosterona en dihidrotestosterona sin aumentar la secreción de LH ni FSH, reduciendo los niveles de dihidrotestosterona en plasma y tejido prostático en más de un 90%.

51.- El diagnóstico mas probable en un hombre de 60 años , con antecedentes de tabaquismo positivo de mas de 30 años de evolución , que presenta de manera brusca un síndrome miccional irritativo con tacto rectal normal, flujo urinario no obstructivo, ecografía reno-vésico-prostática normal, sedimento urinario con microhematuria y urocultivo negativo, cistoscopia normal y citología urinaria con atipias, es:

a) Litiasis uretral. b) Esquistosomiasis. c) Carcinoma in situ vesical. d) Hipernefroma.

Ca vesical. La incidencia es mayor estadísticamente en los fumadores, especialmente de cigarrillos, debido a la acción de la nicotina en el metabolismo del triptofano por bloqueo en su degradación natural, acumulándose compuestos aromáticos con poder cancerígeno. Aumenta más en los grandes fumadores, aquellos que consumen más de 30 cigarrillos al día. Pero aún ante una exposición similar el riesgo varía entre las personas, lo que estaría relacionado con las vías metabólicas propias de cada individuo. Al dejar de fumar el riesgo disminuye lentamente los 20 años siguientes, pero nunca se llega al nivel basal de riesgo.

El cáncer de vejiga es habitualmente de comienzo silencioso y la mayoría de las veces el diagnóstico se plantea por la presencia de hematuria macroscópica, el síntoma más característico. En otras oportunidades puede existir una hematuria microscópica reiterada y síntomas irritativos como disuria, urgencia miccional y polaquiuria. Clásicamente se ha establecido que se puede encontrar este cáncer en aproximadamente el 10% de los pacientes con hematuria.

En el estudio del paciente con hematuria, especialmente en la edad de más riego, se debe solicitar un examen de orina con urocultivo, una ecotomografía renal y pelviana y/o una pielografía de eliminación. El examen de orina nos permite estudiar la existencia de una infección urinaria y también llevar a plantear otras alternativas diagnósticas. El estudio de

imagen superior de ecotomografía y pielografía permite una evaluación completa de la vía urinaria, especialmente alta, donde por ser ésta una enfermedad que compromete todo el urotelio, se puede encontrar algún otro foco tumoral alto o existir dilataciones pieloureterales como consecuencia de obstrucciones que el crecimiento del tumor puede producir.

Posteriormente se debe efectuar una cistoscopía, el examen clásico y aún fundamental para el diagnóstico de esta patología. La cistoscopía permite la visualización completa de la vejiga, a través de la cual se evalúa el tumor en cuanto a su tamaño, número y ubicación y si es pediculado o sésil.

Durante la cistoscopía es importante tomar muestras de biopsia adecuadas, tanto para clasificar el tumor como determinar el grado de infiltración de la pared vesical. En muchas ocasiones cuando se presentan tumores pequeños o medianos este procedimiento es diagnóstico y terapéutico a la vez, ya que la muestra que se extrae es la totalidad del tumor. Para completar el estudio en los pacientes en que se sospecha un tumor invasor, se debe realizar una TAC de abdomen alto y pelviano. Este examen se realiza para determinar la extensión perivesical, para lo cual tiene una sensibilidad del 83% y especificidad del 82%. También nos puede informar de eventuales compromisos ganglionares o a más distancia. No tiene mayor utilidad en los tumores superficiales ni para determinar la invasión muscular de la pared vesical.

Otra posibilidad que en forma excepcional puede utilizarse es la resonancia magnética, pero no tiene ventajas significativas sobre un estudio de TAC, excepto que podría ayudar a definir la extensión en capa muscular vesical.

Referencias 1. 2.

1984

Benson R.C. Endoscopic managemente of bladder tumor. Urol.Cl.North Am.11,637,

Boyd S.D., Skinner D.G. Quality of life of urinary diversion patients.J. of Urology 138,1836, 1989. 3. Bricker E.M. Bladder subsstitution after pelvic evisceration. Cli. North Am. 3o,1511,1950.

4.

1990

Catalona W.J., Ratliff T.L.BCG and superficial bladder cancer. Surgannu. 22,362,

5.

Catalona W.J. Urotelial tumors of the urinary tract. Campbell's Urology. Sounders Companym, Phyladelphia, 1994. 6. Hautmann, Hofstteter F., Franck F. Endoscopic neodyme Yag laser application in bladder tumors.Eur.Urol.7.278,1981. 7. Kock N.G. Ghoneim M.A. Urinary diversion via continent ileal reservoir.J. of Urol. 141,1111, 1989 .

8.

Goodwin W.E. Harris >A.P. Open transcolonic ureterointestinal anastomosis. Surg.Ginecol.Ostetr. 97:295,1953.

52.-Al realizar la exploración clínica y colocar un diapasón que está vibrando frente al conducto auditivo del oído que queremos explorar (conducción aérea) y apoyando después sobre la mastoides (conducción ósea), podemos de modo sencillo y en la consulta, distinguir entre sordera nerviosa (alteración en la cóclea o nervio auditivo) y sordera de conducción (trastorno en el sistema de transmisión tímpano-osicular). ¿Cuál de estas afirmaciones es correcta para un paciente que presenta una sordera de conducción?

a) La percepción del sonido es igual por vía aérea que por vía ósea. b) La percepción del sonido es mejor por vía aérea que por vía ósea. c) La percepción del sonido es mejor por vía ósea que por vía aérea. d) La percepción del sonido es indistinguible tanto por vía aérea como ósea.

MEDICIÓN CLÍNICA DE LA AUDICIÓN La valoración audiológica mínima debe incluir la determinación de los umbrales de conducción aérea y ósea, el umbral de recepción y la discriminación del lenguaje, una timpanometría y pruebas de reflejos acústicos, que incluyan la prueba de deterioro de los reflejos. La información obtenida por medio de estas técnicas permite determinar si hace falta una mayor diferenciación entre la sordera neural y la sensorial. La audición por conducción aérea se valora presentando un estímulo acústico mediante auricular o altavoces. Una sordera o elevación del umbral de audición detectada por esta prueba se puede deber a defectos en cualquier parte del aparato auditivo: pabellón auricular, conducto auditivo, oído medio o interno, VIII par craneal o vías auditivas centrales. La audición por conducción ósea se valora colocando una fuente sonora (el vibrador de un audiómetro o un diapasón) en contacto con la cabeza. El sonido produce una vibración a

través del cráneo, que alcanza las paredes óseas de la cóclea y estimula directamente el oído interno. La audición por conducción ósea no atraviesa los oídos externos y medio y permite valorar la integridad del oído interno, del octavo nervio craneal y de las vías auditivas centrales. Si aumenta el umbral de conducción del aire y el umbral de conducción ósea es normal, la sordera es de conducción, mientras que si ambos umbrales aumentan por igual es de tipo neurosensorial. Existen algunas formas de sordera mixtas con componente neurosensorial y de conducción, en las que aumentan ambos umbrales, aunque el de la conducción aérea es más significativo. Manual Merck 10. Edición en Español

Editors of The Merck Manual

Robert S. Porter, MD, Editor-in-chief Justin L. Kaplan, MD, Senior Assistant Editor Editorial Board of The Merck Manual

53.- Las pruebas de Rinne negativo es propio de las sorderas de:

a) Percepción. b) Transmisión. c) Tumores del acústico. d) Meniere.

TEST DE RINNE: Esta prueba consiste en la comparación de la audición por la vía aérea y vía ósea en un mismo oído para efectuar una aproximación inicial de la localización de la causa de la hipoacusia. Se coloca el mango del diapasón cuya frecuencia se estudia en la apófisis mastoides (vía ósea), y cuando su vibración deja de ser percibida por el paciente se colocan sus ramas unos 15-20mm frente al conducto auditivo externo (vía aérea) hasta que el paciente deja igualmente de oírlo. Los hallazgos detectables están en función del trastorno audiológico existente. Rinne positivo: La audición por vía aérea dura unos 15 segundos más que la percepción del sonido por vía ósea. Traduce normalidad de ese oído.

Rinne negativo: La percepción por vía ósea es mayor que por vía aérea. Implica un trastorno a lo largo de esta vía, bien el conducto auditivo externo, bien la membrana timpánica, bien la cadena de huesecillos.

Manual Merck 10. Edición en Español

Editors of The Merck Manual

Robert S. Porter, MD, Editor-in-chief Justin L. Kaplan, MD, Senior Assistant Editor Editorial Board of The Merck Manual

54.- La patología rara mas frecuente causada por complicación en otitis media aguda es: a) Petrositis. b) Meningitis. c) Osteomielitis. d) Mastoiditis.

La mastoiditis es la complicación que con más frecuencia aparece en las otitis medias supuradas agudas y crónicas; esta resuelve generalmente con tratamiento médico generalmente y evoluciona satisfactoriamente, pero el caso que nos ocupa evolucionó hacia el absceso mastoideo por esta razón y la importancia de la misma decidimos mostrar este paciente con dicha enfermedad, a consecuencia de una otitis media crónica supurada osteítica colosteatomatosa, evolución de la misma y tratamiento quirúrgico realizado. MASTOIDITIS Es la osteítis de la apófisis mastoides manifestada exteriormente. Solo cuando el proceso inflamatorio en el interior de hueso se exterioriza y manifiesta mediante síntomas clínicos se diagnostica mastoiditis. Es la complicación de origen otógeno más frecuente y puede manifestarse aislada o asociada a otras complicaciones intra o extratemporales. Es más frecuente en la infancia y adolescencia si bien puede afectar a personas de todas las edades. 36-42 % de todos los casos de mastoiditis aguda se producen en menores de 2 años, lo más frecuente es que aparezcan entre los 6 meses y 3 años.

Referencias 1. Roddy MG, Glazier SS, Agrawal D. Pediatric mastoiditis in the pneumococcal conjugate vaccine era: symptom duration guides empiric antimicrobial therapy. Pediatr Emerg Care. 2007 Nov; 23(11):779-84. 2. Kvaerner KJ, Bentdal Y, Karevold G. Acute mastoiditis in Norway: no evidence for an increase. Int J Pediatr Otorhinolaryngol. 2007 Oct; 71(10):1579-83. 3. Palma S, Fiumana E, Borgonzoni M, Bovo R, Rosignoli M, Martini A. Mastoiditis in children: the "Ferrara" experience. J Pediatr Otorhinolaryngol. 2007 Nov; 71(11):1663-9. 4. Stähelin-Massik J, Podvinec M, Jakscha J, Rüst ON, Greisser J, Moschopulos M, Gnehm HE. Mastoiditis in children: a prospective, observational study comparing clinical presentation, microbiology, computed tomography, surgical findings and histology. Eur J Pediatr. 2007 Aug 1.

5. Glynn F, Osman L, Colreavy M, Rowley H, Dwyer TP, Blayney A. Acute mastoiditis in children: presentation and long term consequences. J Laryngol Otol. 2007 Jul 19;:1-5. 6. Benito MB, Gorricho BP. Acute mastoiditis: increase in the incidence and complications. Int J Pediatr Otorhinolaryngol. 2007 Jul; 71(7):1007-11.

55.- Un trabajador limpia vidrios con antecedente de fractura vertebral de C6 y fractura del extremo distal del radio derecho tras caída de un tercer piso se presenta a consulta, refiere dolor, limitación de la movilidad y hormigueo en los dedos 2º y 3º de la mano derecha, de predominio nocturno, que mejora durante el día. ¿Cuál de los siguientes diagnósticos es el más probable?: a) Síndrome del túnel del carpo. b) Radiculalgia C5-C6. c) Distrofia simpático refleja. d) Artrosis postraumática del carpo.

El Síndrome del Túnel Carpiano (STC) es una patología que afecta a la mano, provocada por una presión sobre el nervio mediano a nivel de la muñeca. Esto provoca síntomas como adormecimiento y hormigueos en la mano (especialmente en los dedos pulgar, índice, corazón y mitad del anular). Puede existir dolor, que puede estar limitado a la mano y muñeca, pero que en algunas ocasiones se irradia hacia el antebrazo. El STC con frecuencia

despierta al paciente por la noche, y los síntomas pueden aparecer con actividades como conducir un vehículo, escribir, u otros ejercicios que suponen una utilización significativa de la mano. En el síndrome del túnel del carpo avanzado, puede producirse una pérdida de fuerza y una disminución de la masa muscular en la base del pulgar. CAUSA El túnel del carpo es un canal formado por los huesos de la muñeca y un ligamento (el ligamento transverso del carpo) situado en la cara palmar de la muñeca. Por este túnel transcurren todos los tendones que flexionan la muñeca y los dedos, y el nervio (nervio mediano) que recoge la sensibilidad del pulgar, índice, corazón y parte del anular y moviliza los músculos de la base del pulgar. Algunas personas nacen con túneles estrechos y por tanto están predispuestas a problemas de presión sobre el nervio. La utilización vigorosa de la mano, que conduce a una tendinitis de los tendones que flexionan el pulgar y los demás dedos, también puede conducir a un síndrome del túnel del carpo a través del engrosamiento de las vainas tendinosas. Las vainas engrosadas "rellenan" el túnel presionando sobre el nervio. Las personas con artritis reumatoide, hipotiroidismo, diabetes, amiloidosis, insuficiencia renal y algunos otros problemas médicos están más predispuestas a padecer este síndrome.

1.

2. 3. 4. 5.

Arthroshi I, Gummenson C, Johonsson R, Ornstein E, Ranstam J, Rossen I. Prevalence of carpal tunnel syndrome in a general population. JAMA. 1999;282:1538 Periódico El Público, 17 de octubre del 2007,pag 26 Durkan, JA. The carpal compression test: an instrumental device for diagnostic carpal tunnel syndrome. Lancet. 1990;335:393-5. Marshall S, Tardif G. Injection local of steroids in the carpal syndrome. Cochrane Data Base of Systematic Reviews. 2005. Issue 5. Verdugo RJ, Salinas RS, Castillo J, Cea JG. Tratamiento quirúrgico versus tratamiento no quirúrgico para el síndrome del túnel carpiano. Cochrane Data Base. 2005

56.- La presencia de una pigmentación parduzca en los grandes pliegues (cuello, axilas, ingles) con hiperqueratosis, plegamiento y engrosamiento aterciopelado de la piel, se denomina: a) Acantosis nigricans. b) Tinea nigra. c) Psoriasis invers. d) Ictiosis simple.

La acantosis nigricans es una enfermedad rara de la piel, caracterizada por la presencia de hiperqueratosis e hiperpigmentación (lesiones de color gris - parduzco y engrosadas, que dan un aspecto verrucoso y superficie aterciopelada) en los pliegues cutáneos perianales y de las axilas. La acantosis nigricans es un trastorno pigmentario, debido a una hiperinsulinemia secudario a la resistencia de las células del cuerpo en contra de la insulina, tal como se presenta en la diabetes tipo II el cual se acompaña en esta situación en general de signos de hiperandrogenismo (hirsutismo, acné y oligomenorrea en las mujeres) siendo en estos casos una resistencia grave a la insulina,1 el Síndrome Metabólico y en la obesidad.2 Al parecer, la incrementada concentración de insulina activa o estimula a ciertos receptores celulares promotores de la proliferación de células de la piel. Algunas drogas como los anticonceptivos orales, linfomas y algunos canceres pueden causar casos de acantosis pigmentaria.3 Se conocen cuatro tipos de acantosis nigricans: •

El síndrome de Miescher, que es una forma benigna y hereditaria; está asociada con la obesidad afectando a 2/3 de los adolescentes que superan el 200% de su peso ideal.4



El síndrome de Gougerot Carteaud, que es también una forma benigna y posiblemente hereditaria, pero que se da en mujeres jóvenes; La pseudoacantosis nigricans que es una forma juvenil benigna que se asocia a la obesidad y alteraciones endocrinológicas. La acantosis maligna, relativamente infrecuente, se forma en el adulto asociado con frecuencia a un tumor maligno interno, sobre todo del tubo digestivo.

• •

Referencias 1. Harrison Tratado de Medicina interna 17ª ed VOLII pag 2282 2. Aaron S. Katz, et al. Acantosis nigricans en pacientes obesos: formas de presentación e implicaciones para la prevención de la enfermedad vascular aterosclerótica. Dermatology Online Journal. Volume 6, Number 1. [1] 3. MedlinePlus - Acantosis Pigmentaria. [2] 4. ActualidadDermatol.com - Acantosis nigricans. [3]

57.- El mecanismo fisiopatológico por el que el desarrolla choque distributivo es por:

a) Alteraciones en la contractilidad cardiaca. b) Alteraciones en las resistencia vasculares sistémicas. c) Alteraciones ácido-base. d) Alteraciones de catecolaminas.

Es un síndrome resultante de hipoperfusión e hipoxia tisular sistémicos. Desde el punto de

vista fisiopatológico se clasifica en cuatro tipos mayores: hipovolémico, cardiogénico, obstructivo y distributivo. Es posible observar formas puras de cada uno de ellos, pero es frecuente ver distintas combinaciones.

SHOCK DISTRIBUTIVO: 1. Sepsis severa. 2. Anafilaxia: penicilinas y otros antibióticos, algunos antiinflamatorios como los salicilatos, narcoanalgésicos y algunos anestésicos locales y generales y agentes para ayuda diagnóstica como medios de contraste. 3. Neurogénico: bloqueo de los mecanismos de regulación cardiovascular por daño medular, disautonomía, neuropatías periféricas. 4. Medicamentoso: sedantes, vasodilatadores.

Shock Distributivo Determinado por la disminución de la resistencia vascular sistémica provocando la hipotensión correspondiente.

Bibliografía recomendada Revisión Advanced Trauma Life Support® (ATLS®)

58.- Los tumores malignos que surgen en tejidos mesenquimales se les denomina:

a) Hamartomas. b) Carcinomas. c) Sarcomas. d) Teratomas.

Neoplasias malignas mesenquimatosas. Los tumores malignos que nacen en los tejidos mesenquimales suelen denominarse sarcomas, del griego sar (carnoso), porque en general poseen muy poco estroma conjuntivo y en consecuencia su estroma es blando. El esquema utilizado para su denominación es el mismo que en las neoplasias benignas con la utilización del sufijo sarcoma ejemplo, fibrosarcoma, condrosarcoma y leiomiosarcoma. Tejido de origen

Benigno

Maligno

Tejido fibroso

fibroma

fibrosarcoma

Tejido óseo

osteoma

osteosarcoma

Tejido cartilaginoso

condroma

condrosarcoma

Músculo liso

leiomioma

leiomiosarcoma

rabdomioma

rabdomiosarcoma

lipoma

liposarcoma

Vasos sanguíneos

hemangioma

hemangiosarcoma

Linfáticos

linfangioma

linfangiosarcoma

Músculo estriado Tejido adiposo

Hay excepciones como las neoplasias linfoides malignas que se denominan Linfomas. Las originadas en los melanocitos: Melanomas.

59.- Micosis superficial que afecta el estrato córneo y se manifiesta con lesiones discrómicas con descamación fina: a) b) c) d)

Tinea pedis Tinea corporis Pitiriasis versicolor Exofialosis

La pitiriasis versicolor (PV) es una micosis superficial crónica, caracterizada por máculas hipo o hiperpigmentadas levemente descamativas. El término “versicolor” se refiere al color variable de las lesiones que pueden ser café, rosadas o blancas. Esta enfermedad tiene distribución mundial y es uno de los desórdenes más comunes en la pigmentación de la piel. La mayor prevalencia y el mayor porcentaje de recurrencias se observan en áreas cálidas y húmedas.

Eichstedt, en 1846, fue el primero en reconocer la naturaleza fúngica de la PV, al describir un hongo asociado a esta afección. Sin embargo, el género Malassezia, con Malassezia fufur (M. furfur) como especie tipo, fue creado por Baillon medio siglo después, en 1889, en honor a Luis Malassez. Pitiriasis versicolor, a veces denominada Tinea versicolor, es uno de los trastornos de pigmentación más comunes en el mundo. Es una micosis crónica, leve y usualmente asintomática. Se produce por el crecimiento de la levadura lipofílica Malassezia furfur en la capa más superficial de la piel o estrato córneo. La infección se caracteriza por el desarrollo de manchas o máculas lenticulares de color café, rosado o blanco, levemente descamativas, en tronco y brazos.

BIBLIOGRAFÍA 1. Ashbee HR, Evans E.G. Inmunology of Diseases Associated with Malassezia species. Clin Microbiol Rew. 2002;21-57. 2. Crespo Erchiga V, Delgado Florencio V. Malassezia species in skin diseases. Curr Opinión Infect Dis, 2002, 15:133-142. 3. Guèho E, Boekhout T, Ashbee HR, Guillot J, Van Belckum A, Faergeman J. The role of Malassezia species in the ecology of human skin and as pathogen. Med Mycol 1998;36 (supp1):220-229. 4. Crespo Erchiga V, Ojeda Martos A, Vera Casaño A, Crespo Erchiga A, Sánchez Fajardo F, Guèho E. Mycology of pityriasis versicolor. J. Mycol. Med. 1999; 9:143-148.

5. Katoh T, Irimajiri J. Pityriasis versicolor and Malassezia folliculitis. Nippon Ishinkin Gakkai Zasshi. 1999;40:69-71. 6. Gupta AK, Batra R, Bluhm R, Boekhout T, Dawson T. Skin diseases associated with Malassezia species. J Am Acad Dermatol 2004; 51:785-798. 7. Gupta AK, Kohli Y, Faergemann J, Summerbell RC. Epidemiology of the Malassezia yeast associated with pityriasis versicolor in Ontario, Canada. Med Mycol 2001, 39:199-206.

60.- ¿Cuál es el primer dato clínico de desarrollo puberal en los niños?

a) Cambio en la voz.

b) Desarrollo muscular. c) Crecimiento testicular. d) Salida de vello púbico.

Cuando las neuronas productoras de GnRH (hormona liberadora de las gonadotropinas) maduran, empiezan a generarse pulsos de GnRH capaces de liberar LH y FSH. Ambas aumentan el tamaño testicular y se inicia la producción de testosterona, inicialmente en niveles muy bajos. Bashin S, Jameson JL. Trastornos de los testículos y del sistema reproductor masculino. En Jameson JL (ed): Harrison. Endocrinología. 1a ed. Madrid. MacGraw-Hill España, 2006: 17475.

61.- Varón de 31 años de edad que desde hace tiempo tiene episodios de ojo rojo con ausencia de secreción. Refiere leve sensación de cuerpo extraño y una carnosidad que crece y ya alcanzó la córnea. A la exploración se observa un tejido de neoformación nasal que invade 1mm de la córnea.

a) b) c) d)

Carcinoma intraepitelial de conjuntiva. Pingüecula. Nevo conjuntival. Pterigion.

El pterigión constituye una hiperplasia fibrovascular de carácter benigno de la conjuntiva bulbar que invade la córnea,1-4 está clasificado dentro de las degeneraciones no involutivas o tumoraciones epiteliales benignas corneales.1,3,5 Se localiza en la conjuntiva bulbar cerca del limbo corneal en el área interpalpebral, a las 3 y 9 horas. Puede ser unipolar (solo afecta una parte) o bipolar, cuando afecta tanto la parte temporal como la nasal, es más frecuente en el lado nasal. También pueden ser unilaterales o bilaterales. Recibe su nombre por su aspecto de ala pequeña. Presenta una forma triangular, cuyo vértice mira hacia el área pupilar y cuya base se orienta hacia la carúncula en la localización nasal, y hacia el lado temporal en los localizados en el área temporal. Habitualmente muestra un crecimiento horizontal que puede llegar a afectar el eje visual.2,3,7

Actualmente, en los estudios sobre alteraciones ultraestructurales del pterigión, se considera una alteración inflamatoria y proliferativa de la superficie ocular.2 Se plantea que las células madre limbares se modifican con exposición crónica a la luz ultravioleta, por lo que hay una rotura de la barrera limbar que causa la invasión conjuntival del epitelio corneal. Otro mecanismo que se expone relacionado con las reacciones inmunológicas es una reacción de hipersensibilidad tipo I a elementos irritantes exógenos (polvo, viento, etc.) asociada a una inflamación local que causa un incremento en la producción de IgE.3 Un estudio reciente muestra muestra una asociación entre una película lagrimal inestable y el inicio de un pterigión. Referencias bibliográficas 1. Adamis AP, Stark T, Kenyon KR. The manegement of pterygium. Ophtamol Clin North Am. 1990;3(4):611 2. Klinworth GK. Chronic Actinic keratopathy, a condition associated with conjunctival elastosis (pingueculae) and typified by characteristic extracellular concretions. Am J Pathol. 1972;67:32. 3. Mac Kenzie FD, Hirst LW, Battistutta D. Risk analysis in the development of pterygia. Ophthalmology. 1992;99:1056-61. 4. Clear AS, Chirambo MC, Hutt MSR. Solar keratosis, ptert-gium, and squamous cell carcinoma of the conjunctiva in Malawi. Br J Ophthalmol. 1979;63:102-9. 5. Sánchez Thorin JC, Rocha G, Yelin 313. Meta-analysis on the recurrence rales after bare sclera resection with and without mitomycin C use and conjunctival autograft placement in surgery for primary pterygium. Br J Ophthalmol. 1998;82:661-5.

6. Lam DSC, Wong AKK, Fan DSP, et al. Intraoperative mito-mycin C to prevent recurrence of pterygium after excision: a 30-month follow-up study. Ophthalmology. 1998;105:901-5. 7. Tan DTR, Lim ASM, Goh RS, Smith DR. Abnormal ex-pression of the p53 tumor suppressor gene in the conjunc-tiva of patients with pterygium. Am J Ophthalmol. 1997;123:404-5.

62. - La causa principal de insuficiencia arterial aguda es: a) b) c) d)

Ateroembolismo. Traumatismo vascular. Embolismo cardiaco. Ateroesclerosis.

La causa más frecuente de obstrucción arterial aguda es la embolia de la circulación mayor. La principal fuente de émbolos arteriales es el corazón izquierdo (ya sea por trastornos del ritmo cardiaco, infarto agudo de miocardio, etc.) y la mayoría de estos émbolos migra hacia las extremidades inferiores. El lugar donde más frecuentemente asientan dichos émbolos es a nivel de la arteria femoral común, en su bifuración, o bien más distalmente. También son habituales en extremidades superiores, carótidas y, más raramente, en las mesentéricas. Después de la embolia arterial, la trombosis arterial aguda es la causa más frecuente de obstrucción arterial aguda. La trombosis casi siempre se produce por alteraciones previas del endotelio vascular, generalmente de causa arteriosclerótica. La formación de un trombo in situ puede verse favorecida por distintos mecanismos: una estenosis arterial severa, situaciones de hiperviscosidad o hipercoagulabilidad, bajo gasto cardiaco y otras. En estos casos de trombosis arterial hay un segmento especialmente susceptible: la femoral superficial a nivel del anillo de los aductores Otras causas menos frecuentes son el arteriospasmo, los traumatismos, la compresión extrínseca y el aneurisma de aorta diseeante.

Fuentes posibles de Embolias Arteriales • • • • •

Valvulopatía mitral o aórtica con dilatación auricular izquierda y fibrilación auricular. Trombo adherido a la pared ventricular izquierda, dañada por infarto del miocardio. Prótesis valvulares cardíacas. Endocarditis bacteriana. Tumores intracardíacos (por ej.: mixoma)

• •

Trombo adherido a la pared aórtica enferma (por ej.:aneurisma o ateroma ulcerado). Trombo de origen venoso en paciente con defecto del tabique interauricular (embolia paradojal)

El 50% de las embolías afecta a los vasos de las extremidades inferiores, y menos del 15% compromete las extremidades superiores. En la tabla III se detallan algunas causas de trombosis arterial. Tabla III. Causas de Trombosis Arterial • • • • • •

Daño o injuria de la pared arterial. Ateroesclerosis Obliterante Enfermedades del Colágeno Enfermedades Mieloproliferativas Disproteinemias Trombofilias

BIBLIOGRAFIA Schwartz .S, et al ; PRINCIPIOS DE CIRUGÍA: Séptima edición , Vol I, pp; 1017-1024; McGraw – Hill Interamericana; 2000. Veith .F, Hobson .R, Williams .R, Wilson .S ; VASCULAR SURGERY PRINCIPLES AND PRACTICE: Second Edition, McGraw-Hill; 1994 Alvarez Fernández J. Gutiérrez Julián J. Menéndez Herrero M.; ISQUEMIA AGUDA DE LAS EXTREMIDADES; Hospital Central De Asturias. Vaquero F. Uriach L.; ISQUEMIAS AGUDAS; 1994. Sabiston. CIRUGIA ; 2000 Scribner R. Brown W. Tawes R.; DECISION MAKING IN VASCULAR SURGERY; B.C. Decker Inc. Toronto – Philadelphia. 1987.

Lazar G. Michael M. Keith O. Gerald Z. Keith L.; ESSENTIALS OF SURGERY STIENTIFIC PRINCIPLES AND PRACTICE.; Editorial Lippincott – Raven; Washington Square – Philadelphia. 1997.

63.- ¿Cuál es una contraindicación absoluta para iniciar alimentación enteral de cualquier tipo? a) Pancreatitis. b) Isquemia intestinal. c) Insuficiencia cardiaca congestiva. d) Enfermedad inflamatoria intestinal.

Humberto Arenas Marques, Roberto Anaya Prado. Nutrición enteral y parenteral. McGraw Hill 2007:261-270 La isquemia intestinal es una contraindicación absoluta para la administración de nutrición enteral debido a que la distensión y la presencia de nutrientes favorecerán la presencia de isquemia intestinal y mayor daño.

64.-. Paciente con prurito y lagrimeo en ambos ojos, hiperemia conjuntival, fotofobia, exudado más o menos viscoso y formación papilar en la conjuntiva tarsal. ¿El diagnóstico probable es?

a) Queratoconjuntivitis seca. b) Conjuntivitis alérgica. c) Conjuntivitis bacteriana. d) Conjuntivitis vírica.

Conjuntivitis alérgica. Condición inflamatoria ocular bilateral, crónica y recurrente, que se presenta predominantemente en la infancia y con mayor incidencia en el sexo masculino. Las exacerbaciones se asocian a ciertas estaciones del año (donde el nombre de "primaveral") en las cuales se supone que existe una mayor cantidad de alergenos en el medio ambiente; por ejemplo, el polen de las plantas. Estos pacientes frecuentemente presentan historia personal o familiar de atopia. El síntoma principal es el prurito ocular intenso, además de secreción mucoacuosa matutina, fotofobia severa y lagrimeo. Clínicamente se observa intensa hiperemia y quemosis conjuntival bulbar, presencia de papilas gigantes en la

conjuntiva tarsal, secreción mucoide abundante, ocasionalmente se observan infiltrados eosinófilos a nivel de limbo que se conocen como "puntos de Trantas" y en etapas crónicas se aprecia acúmulo de pigmento en conjuntiva bulbar y opacificación corneal secundaria a queratopatía punteada. Es frecuente que estos pacientes presenten una sobreinfección debido al contacto de las manos con estructuras oculares por el prurito incontrolable. Se

ha demostrado presencia de eosinófilos e inmunoglobulina IgE; sin embargo, el diagnóstico es esencialmente clínico. El tratamiento se divide en medidas preventivas, paliativas y antiinflamatorias. Las medidas preventivas se orientan a evitar la exposición directa al sol, utilización de visera y lentes oscuros; el tratamiento sintomático se basa en la aplicación de fomentos fríos, lubricantes y vasoconstrictores tópicos y el uso de antihistamínicos orales. Como antiinflamatorios tópicos en etapas agudas se emplean cursos cortos de esteroides. Los estabilizadores de células cebadas (cromoglicato de sodio) y antihistamínicos tópicos tienen efecto a largo plazo. Generalmente estos pacientes presentan mejoría importante en la intensidad y frecuencia de los episodios al alcanzar la adolescencia.

Referencias Bibliográficas: 1.

Miller Stephen JH. Parsons’ Diseases of the Eye. 18th ed. Edinburgh, London, Melbourne and New York: Churchil Livigstone; 1990. 2. Sánchez Salorio M, Rodríguez Ares T, Alio y Sanz JL, Pita D, Fontanela JR, Grau M et al. Conjuntivitis, Edika-Med S.A. Barcelona: CIBA VISION, 1992. 2. Foulks GN, Pavan-Langston D. Cornea and External Disease. In: Pavan-Langston D, editor. Manual of Ocular Diagnosis and Therapy. 2nd ed. Boston: Little, Brown & Co. 1991. p. 100-102.

65.- Complicación que puede surgir por la enfermedad o por el tratamiento en casos de tuberculosis urinaria:

a) b) c) d)

Gangrena gaseosa. Divertículos ureterales. Cáncer de urotelio. Estenosis ureteral.

Durante la fase de diseminación hematógena (primoinfección) se produce siembra de bacilos en ambos riñones en el 90% de los casos, sin embargo, la enfermedad clínica generalmente es unilateral. El período de latencia entre la “siembra” y la enfermedad

clínica oscila entre 10 y 40 años, afectando principalmente a pacientes por debajo de los 50 años. La lesión inicial microscópica se localiza en los glomérulos en forma de granulomas microscópicos. Al avanzar la enfermedad se produce afectación más distal hasta la aparición de una papilitis necrotizante, momento en el cual puede ya existir paso de bacilos a la vía excretora, donde por procesos inflamatorios ocasionará estenosis a nivel de los infundíbulos caliciales, pelvis y uréter, con hidronefrosis secundaria.

Manual CTO 7ª edición, Urología, pág5.

66.- Al explorar a un paciente masculino de 40 años de edad quien presenta una otoscopía sin alteraciones, una hipoacusia derecha, con Rinne negativo derecho y positivo izquierdo y Weber con lateralización al lado derecho. ¿Usted piensa en?

a) Otitis serosa. b) Neuronitis vestibular). c) Hipoacusia súbita. d) Otosclerosis.

OTOSCLEROSIS

INTRODUCCION: OSTEODISTROFIA DE LA CAPSULA OTICA DEL LABERINTO. CLINICA: • HIPOACUSIA: • HIPOACUSIA PROGRESIVA TRANSMISIVA UNILATERAL , EN LA CUAL LA PACIENTE ENTIENDE MEJOR EN AMBIENTES RUIDOSOS.(PARACUSIA DE WILLIS) • PROGRESIVA: AVANZA CON EL TIEMPO, EMBARAZOS. • TRANSMISIVA EN INICIO. LUEGO PUEDE HACERSE NEUROSENSORIAL. • UNILATERAL EN INICIO. LUEGO BILATERAL EN > DEL 75% • PARACUSIA DE WEBER: NO OYE CON MASTICACION. • VERTIGO : 10-20%. . SENSACION DE “BORRACHERA” OCASIONAL. • ACUFENO: 75% . DE TONO GRAVE. MUY MOLESTOS. (ANSIOLITICOS) • ALTS PSIQUICAS: IRRITABILIDAD, INTROVERSION,.HIPOCONDRIACOS,...NEUROSIS.

EXPLORACION: • OTOSCOPIA: NORMAL (MANCHA DE SCHWARTZE 10%) • ACUMETRIA: RINNE NEGATIVO. WEBER LATERALIZADO AL LADO MAS AFECTO SCHWABACH :ALARGADO.(VIA OSEA PACIENTE Y EXPLOR) (TRIADA DE BEZOLD) PR.GELLE: NEGATIVO (DIAPASON EN FRENTE—PRESION CON PERA DE POLITZER EN CAE.) • IMPEDANCIOMETRIA: CURVAS CENTRADAS Y EN OCASIONES ALGO APLANADAS. REFLEJO AUSENTE O INVERTIDO.(NO RECRUITMENT) • TAC: SI VERTIGO. SE VE FOCO OTOESPONGIOTICO. • AUDIOMETRIA: (+AUDIO VERBAL) ESTADIOS: ♣ FORMA TIPICA O DE POLITZER-SIEBENMANN: LA MAS FRECUENTE. PROGRESION LENTA.

1) HIPOACUSIA DE TRANSMISION CON UMBRAL DE 20-40 Dbs,PEOR EN FRECUENCIAS GRAVES. 2) HIPOACUSIA MIXTA: TRANSMISION DE 40-60 DBS Y PERCEPCION DE 20-30DBS (ESCOTOMA EN 2000Hzs-MUESCA DE CARHART). 3) SORDERA CON GRAN COMPONENTE NEUROSENSORIAL DE 6080DBS ♣OTOSCLEROSIS DE LERMOYEZ: HIPOACUSIA TRANSMISIVA PURA. ♣OTOSCLEROSIS COCLEAR DE MANASSE: DESDE INICIO COMPONENTE NEUROSENSORIAL.PURA

Bibliografía • • • • •

House J. Otosclerosis. Otolaryngol Clinics 1993;26(3):323-502. Jerger J. Clinical experience with impedance audiometry. Arch Otolaryngol 1970;92:311. Lempert J. Improvement in hearing in cases of otosclerosis: A new, one stage surgical technique. Arch Otolaryngol 1938;28:42-97. Lippy WH, Schuring AG. Treatment of the inadvertently mobilized footplate. Otolaryngol Head Neck Surg 1973;98:80-81. Meyer S. The effect of stapes surgery on high frequency hearing in patients with otosclerosis Am J Otol 1999;20:36-40.

67.- Femenino de 72 años con hipoacusia bilateral que ha ido progresando en los últimos años. Refiere oír pero no entender, especialmente cuando hay ruido ambiente. La audiometría tonal muestra una hipoacusia neurosensorial bilateral y casi simétrica para ambos oídos por afectación de los tonos agudos. Se comprueba una disminución de la inteligibilidad en el audiograma verbal. ¿Cuál es el diagnóstico más probable?:

a) Presbiacusia. b) Colesteatoma bilateral. c) Insuficiencia vertebrobasilar. d) Enfermedad de Meniere bilateral.

a presbiacusia (alteración de la audición relacionada con el envejecimiento) es la causa más frecuente de hipoacusia perceptiva en los adultos. Los factores que favorecen este trastorno son de tipo genético y ambiental, asociados sobre todo a la exposición al ruido. Existe una variabilidad interindividual en cuanto a la edad de aparición de la presbiacusia y a su evolución. El diagnóstico se establece ante la comprobación de una hipoacusia perceptiva bilateral y simétrica, de predominio en frecuencias agudas. La alteración de la inteligibilidad en la audiometría verbal es un elemento pronóstico. El tratamiento óptimo se basa en la adaptación audioprotésica bilateral y precoz. En algunos casos, la rehabilitación auditiva se favorece con una estrategia multidisciplinaria que incluya la rehabilitación logopédica y un posible apoyo psicológico. En ausencia de mejoría, la edad no representa una contraindicación para el implante de oído medio o coclear. Guerrero Medina, Enrique. Hipoacusia por Ruido. En: Seminario de Salud Ocupacional. . Memorias del III Seminario de Salud Ocupacional CAFAM, 1993. p.6. Enrique, Escuela Colombiana de Medicina, 1993. Notas Técnicas de Prevención: Ruido: vigilancia epidemiológica de los trabajadores expuestos. Instituto Nacional de Seguridad e Higiene en el Trabajo . NTP - 193. 1988. Barcelona : INSHT. p. 2. ISSN-0212-0631. Notas técnicas de Prevención : Hipoacusia laboral por exposición a ruido: Evaluación clínica y diagnóstico. Instituto Nacional de Seguridad e Higiene en el Trabajo (España). NTP - 287. 1991. Barcelona: INSHT. p. 3.

68.- De las pruebas que se citan a continuación, ¿cuál es la más sensible para el diagnóstico del síndrome de Zollinger-Ellison?

a) La detrminación de gastrina basal. b) BAO superior a 15 mEq/hora. c) La estimulación de la gastrina con calcio intravenoso. d) Las biopsias de la mucosa antral.

Criterios Diagnóstico para Síndrome Zollinger-Ellison (ZES) Los criterios diagnóstico para ZES incluyen los siguientes: 1.

2. 3. 4. 5.

Niveles elevados de secreción ácida basal (BAO), mayor de 15 mEq en pacientes no operados y mayor de 5 mEq si previamente se realizó una cirugía para reducir la secreción ácida; Nivel elevado de gastrina sérica en ayunas (>100 pg/mL hasta 1994, >200 pg/mL desde 1994); Resultados anormales en las pruebas de estimulación con secretina (un incremento >200 pg/mL postinyección) o con calcio (un incremento >395 pg/mL); Confirmación histológica positiva de gastrinoma; o Una combinación de estos criterios.

Las pruebas más comúnmente utilizadas para establecer el diagnóstico de ZES son la concentración sérica de gastrina en ayunas y la evaluación de BAO. La medición del pH gástrico es importante para excluir aclorhidria como una causa secundaria de hipergastrinemia. Los pacientes que ingieren inhibidores de la bomba de protones (PPIs), aquellos quienes tienen resección masiva de intestino delgado, o aquellos quienes tienen insuficiencia renal, hiperplasia de células G, u obstrucción pilórica pueden tener niveles de gastrina entre 150 y 1000 pg/mL Por lo tanto, para pacientes con sospecha de ZES con una prueba dudosa de concentración de gastrina sérica en ayunas, una prueba de estimulación con secretina debería ser realizada.

Bibliografía: 1.

Gibril F, Schumann M, Pace A, Jensen RT. Multiple endocrine neoplasia type 1 and Zollinger-Ellison syndrome: a prospective study of 107 cases and comparison with 1009 cases from the literature. Medicine (Baltimore). 2004;83:43-83. 2. Jensen RT; Gastrinomas: advances in diagnosis and management. Neuroendocrinology. 2004;80 Suppl 1:23-7.

69.- Se trata de paciente de 17 años de edad que es enviado para valoración por ortopedia, el paciente refiere dolor cervical bien delimitado el cual no mejora con AINES, en la radiografía se aprecia una imagen radiolúcida y expansiva de 4 cm de diámetro en el pedículo de la vértebra T12. ¿Cuál de las siguientes lesiones tumorales será la más probable?:

a) Encondroma. c) Tumor de Ewing. b) Osteoblastoma. d) Metástasis de cáncer de pulmón.

Osteoteoblastoma Benigno Llamado también osteoide gigante; conformado por células de naturaleza osteoblástica, formador de tejido óseo y osteoide, bastante vascular, generalmente de más de 2 cm de diámetro; carece de dolor típico y de hueso reactivo y tiene un cierto potencial de crecimiento. No es frecuente; más en el sexo masculino, entre 10 y 25 años; evidente preferencia por la columna vertebral, incluyendo el sacro, fémur, tibia, huesos tubulares cortos de manos y pies. • Clínica El dolor, generalmente local, es el síntoma cardinal, sin tener la intensidad típica de las algias del osteoma osteoide; es producido por la compresión por el tumor, ya sea de médula o nervios radiculares y ocasiona a veces paraparesia o paraplejia, otras veces escoliosis, espasmo muscular y síntomas neurológicos. • A RX

Zona lítica,radiolucida de 2 a 10 cm de diámetro rodeada por capa de osteoesclerosis densa, hueso expandido y engrosado; cuando están localizados en tejido esponjoso hay ausencia habitual de osteoesclerosis perifocal.

• Anatomía patológica Macroscópica: bien limitado, hemorrágico, granuloso y friable; el componente osteoide es de grado de calcificación variable; el hueso adyacente no muestra osteoesclerosis, salvo en los huesos diafisarios, en que hay zona de hiperostosis. Microscópica: Los osteoblastos con núcleos regulares poco cromatínicos y con abundante protoplasma producen trabéculas entrelazadas o discretos islotes de sustancia osteoide o tejido óseo; hay amplia variación en la tipología microscópica, produciendo confusión con los tumores a células gigantes, osteomas osteoides, osteoma y quiste óseo aneurismático; hay reabsorción osteoclástica y también hueso más maduro en reconstrucción; nunca se observa formación de cartílago; no hay polimorfismo celular, aunque en lesiones jóvenes a veces se observan figuras mitóticas que pueden confundir con sarcomas. Al tener una histología similar al osteoma osteoide, nadie ha observado que un osteoma osteoide crezca o que un osteoblastoma haya tenido de inicio un osteoma osteoide. A veces maligniza. • Tratamiento El de elección es curetaje de toda la lesión, seguido de colocación de injertos si fuese necesario; si la localización lo permite, la escisión en bloque es aconsejable. No es aconsejable la radioterapia.

Tumores Óseos en general. Tumores Benignos de los Huesos

Dr. Luis Julio Huaroto Rosa-Pérez

Cirugía: II cirugía ortopédica y traumatología Autor: Universidad Nacional Mayor de San Marcos (Lima). Facultad de Medicina. Escuela Académico Profesional de Medicina Humana. Departamento Académico de Cirugía Publicación: Lima: UNMSM, 2000 Descripción: 407 p. : il., tablas, fotos ; 24 cm. Serie: (Cirugía; 2) ISBN: 9972-46-102-5. Otros autores: Salaverry García, Oswaldo, 1959-, ed. Tema: Traumatología; Ortopedia.

70.- Ingresa al servicio de urgencias masculino de 47 años con hemorragia digestiva alta. No hay antecedentes de consumo de AINE. La endoscopia revela úlcera gástrica en incisura angularis con un punto de hematina y mínimos restos de sangre oscura en el estómago. Se realizan biopsias del margen de la úlcera y una biopsia antral para prueba rápida de ureasa positiva. ¿Cuál de las siguientes actitudes es la más correcta?

a) Iniciar con sucralfato y Ranitidina 150 mg/día,tratamiento anti-Helicobacter pylori si la histología confirma la presencia del germen. b) Se debe indicar anti-Helicobacter pylori durante 14 días, seguido de un antisecretor hasta que se confirme la erradicación del germen. c) Esclerosis endoscópica de la úlcera seguida de tratamiento con omeprazol, 20 mg/día durante 28 días. d) Iniciar tratamiento con Omeprazol 40mg/día durante 1 mes.

El Helicobacter pylori (HP) es una bacteria microaerófila, gramnegativa, de crecimiento lento y forma helicoidal con abundantes flagelos. Fue descubierta por dos médicos autralianos. Robin Warren y Barry Marshall; trabajando en colaboración, detectaron que este microorganismo se encontraba en casi todos los pacientes con inflamación gástrica, úlcera duodenal o gástrica. Basándose en

estos resultados propusieron que HP estaba implicado en la etiología de estas enfermedades. Antes de 1982, se pensaba que la mayor causa de la úlcera péptica era el

estrés y el estilo de vida. Ahora se sabe que HP está implicado en más del 90% de las úlceras duodenales y hasta el 80% de las úlceras gástricas. Gracias a los descubrimientos de Marshall y Warren, la úlcera péptica no es una enfermedad crónica sino que puede ser curada con una pauta de tratamiento con antibióticos y con inhibidores de la secreción ácida. Afecta al 50 % de la población mundial, ha sido identificado como el agente causal de la úlcera péptica y se ha clasificado además como carcinógeno tipo I. Como resultado de su interferencia con la secreción de ácido por el estómago, esta bacteria es capaz de generar deficiencias en la absorción de nutrientes y vincularse con la aparición de manifestaciones carenciales o con el agente causal de enfermedades crónicas.

El objetivo del tratamiento médico será promover la cicatrización de la úlcera para prevenir la recurrencia de la hemorragia. Así, la ranitidina puede prevenir la úlcera duodenal en pacientes tomadores de AINES, pero es ineficaz en la prevención de la ulcera gástrica. Por el contrario, la famotidina protege la mucosa gástrica contra tratamientos cortos de aspirina o naproxeno. Si se demuestra que existe infección por H.Pylori estará indicada su erradicación para evitar recurrencias. Amoxicilina + Klaritromicina OD+ esomeprazol o lanzoprazol Erradicación en 14 dias 100% De existir una complicación, como la hemorragia digestiva, los pacientes deberán ser internados, suprimir el aporte oral, efectuar el diagnóstico de hemorragia, determinar la cuantía de la pérdida hemática y realizar un adecuado control de la hemodinamia.

Lahaie RG, Gaudreau C. Helicobacter pylori antibiotic resistance: trends over time. Canadian Journal of Gastroenterology. 2000;14(10):895–899.

Manes G, Balzano A, Iaquinto G, et al. Accuracy of the stool antigen test in the diagnosis of Helicobacter pylori infection before treatment and in patients on omeprazole therapy. Alimentary Pharmacology and Therapeutics. 2001;15(1):73–79. McManus TJ. Helicobacter pylori: an emerging infectious disease. Nurse Practitioner. 2000;25(8):42–46.

71.- Nos encontramos ante una otitis media recurrente cuando se presentan: a) Cuadros repetidos de otitis media que alternan sin periodos de remisión, 3 o más cuadros en 6 meses, o 4 o más cuadros en 1 año. b) Presencia de signos y síntomas de más de 3 meses de duración, con perforación del tímpano. c) Cuadros repetidos de otitis media que alternan con periodos de remisión evidente, 3 o más cuadros en 12 meses, o 4 o más cuadros en 2 años.

d) Cuadros repetidos de otitis media que alternan con periodos de remisión evidente, 3 o más cuadros en 6 meses, o 4 o más cuadros en 1 año.

La definición establecida para recurrencia requiere: 1) remisión evidente entre los cuados y 2) una frecuencia mínima de 3 o más cuadros en 6 meses, o 4 o más cuadros en un año. González-Saldaña N, Infectología Clínica Pediátrica, 7ª edición, páginas 63-98.

72.- ¿Cuál de las siguientes es la indicación por excelencia para iniciar nutrición parenteral total y absoluta en un enfermo? a) b) c) d)

Colitis Ulcerativa Crónica Inespecífica (CUCI) Resección intestinal posquirúrgica. Oclusión intestinal. Cáncer de colon.

Humberto Arenas Marques, Roberto Anaya Prado. Nutrición enteral y parenteral. McGraw Hill 2007:243-250. La nutrición parenteral total esta indicada en pacientes con presencia de enfermedades tales como pancreatitis, enfermedades inflamatorias intestinales, resecciónes intestinales posquirúrgicas que no tienen posibilidad de alimentación distal a la anastomosis, y cáncer entre otras; pero una de las indicaciones absolutas para la aplicación de nutrición parenteral total es la oclusión intestinal, debido a la imposibilidad de utilizar el tubo digestivo.

73.- Se trata de paciente masculino de 33 años el cual presenta los siguientes datos clínicos, álgia facial, rinorrea posterior , obstrucción nasal, éstos son criterios mayores para el diagnóstico de:

a) b) c) d)

Sinusitis aguda. Faringoamigdalitis estreptocócica. Rinitis vasomotora. Rinitis estacional.

CRITERIOS CLÍNICOS El diagnostico de la sinusitis aguda es clínica, depende en la presencia de por lo menos dos síntomas mayores, o un síntoma mayor y dos menores.

Síntomas mayores: • • • •

Dolor o presión facial. Obstrucción nasal. Rinorrea purulenta. Hiposmia o anosmia.

Síntomas menores: • • • • •

Cefalea. Halitosis. Dolor dental superior. Tos, especialmente en niños. Otalgia o presión en oídos.

La combinación de tres de cada cuatro criterios ha dado una especificidad del 81% y una sensibilidad del 66%. En estudios realizados por Williams y Cols, encontraron en hombres adultos, que la congestión nasal, tos, estornudos tienen sensibilidad del 70% al 72%. El síntoma más específico (93%) es el dolor en senos maxilares. En un estudio en Europa, el indicador más sensible fue el dolor en los dientes (83%). El estándar de oro para el diagnóstico de la sinusitis maxilar es el hallazgo de material purulento a través de la aspiración del seno maxilar. Williams Jr JW, Aguilar C, Cornell J, Chiquette E. Dolor RJ, Makela M, Holleman DR, Simel DL. Antibiotics for acute maxillary sinusitis. Cochrane Database of Systematic Reviews 2003, Issue

74.- Masculino de 42 años con antecedentes de hipertensión arterial sistémica tratado con losartan 50 mg v.o. actualmente controlado, inició con sintomatología urinaria asociado a dolor en fosa renal izquierda, se diagnostica un cálculo de 1cm ubicado en la pelvis renal, el tratamiento ideal para este paciente es :

a) Pielolitomía. b) Plastía ureteropiélica.

c) Litotricia electro hidráulica. d) Litotripsia extracorpórea.

La litotripsia extracorpórea constituye el tratamiento en la actualidad más utilizado para la litiasis urinaria. Las contraindicaciones absolutas para este procedimiento son: embarazo, obstrucción distal y la presencia de infección activa. Presenta como limitación cálculos no localizables debido a su pequeño tamaño (menor 2-5mm) Manual CTO de Medicina y Cirugía. Séptima Edición. McGraw Hill, pgs 1637-1642.

75.- Son características clínicas de la insuficiencia arterial aguda:

a) b) c) d)

Pulsos disminuidos, palidez edema y dolor. Pulsos presentes, edema, cianosis y dolor. Ausencia de pulsos palidez, parestesias. Pulsos ausentes, eritema y edema.

Manifestaciones clínicas: Se presentan las seis P: Dolor (Pain): Se presenta en un 75% de pacientes como el primer síntoma, aunque puede no presentarse, esto debido a una circulación colateral adecuada, neuropatía diabética, o ha una progresión rápida a isquemia avanzada con anestesia inmediata. Parestesias y Parálisis: Las parestesias nos indican anoxia de terminaciones nerviosas sensitivas y motoras. Las parálisis nos hablan de necrosis de músculo estriado. Al comenzar la isquemia los músculos son blandos, con el tiempo se presenta edema y se produce una sensación pastosa en el sitio de la oclusión, luego al presentarse la necrosis los músculos se vuelven rígidos y duros. En ésta última etapa la alteración isquémica es irreversible. Sí las parestesias y parálisis no son aliviadas en el transcurso de 6 a 8 horas, se desarrollara gangrena en la extremidad afectada. Palidez y Poiquilotermia: la reducción del flujo sanguíneo en una extremidad, le confiere el aspecto de pálido y la extremidad se enfría. La piel moteada que no empalidece a la

presióndigital nos indica isquemia irreversible y se debe a extravasación de sangrea la dermis por los capilares rotos. Pulsos periféricos ausentes: Indica una oclusión arterial, al examinar todos los pulsos de una extremidad nos puede ayudar a encontrar la altura de la obstrucción.

BIBLIOGRAFIA Schwartz .S, et al ; PRINCIPIOS DE CIRUGÍA: Séptima edición , Vol I, pp; 1017-1024; McGraw – Hill Interamericana; 2000. Veith .F, Hobson .R, Williams .R, Wilson .S ; VASCULAR SURGERY PRINCIPLES AND PRACTICE: Second Edition, McGraw-Hill; 1994 Álvarez Fernández J. Gutiérrez Julián J. Menéndez Herrero M.; ISQUEMIA AGUDA DE LAS EXTREMIDADES; Hospital Central De Asturias. Vaquero F. Uriach L.; ISQUEMIAS AGUDAS; 1994. Sabiston. CIRUGIA ; 2000 Scribner R. Brown W. Tawes R.; DECISION MAKING IN VASCULAR SURGERY; B.C. Decker Inc. Toronto – Philadelphia. 1987. Lazar G. Michael M. Keith O. Gerald Z. Keith L.; ESSENTIALS OF SURGERY STIENTIFIC PRINCIPLES AND PRACTICE.; Editorial Lippincott – Raven; Washington Square – Philadelphia. 1997.

76.- ¿Las gonadotropinas son hormonas hipofisiarias que regulan la secreción de hormonas ováricas, de qué genero? a) Glucoproteínas. b) Hormonas esteroides. c) Lipoproteínas. d) Decapéptidos.

Glicoproteína, hidratos de C en 2 y 17, le da gran estabilidad y vida media muy larga. Receptores de membrana específicos (hormona proteica). Regula su secreción por mecanismos de retrocontrol largo y corto. Hay efecto bifásico de esteroides (retrocontrol más importante que sobre LH). Las tasas de FSH son en ascenso hasta pico y luego en descenso. 10 miliunidades /ml que se duplica en pico y disminuye en fase postovulatoria más bajas que preovulatoria. El ciclo normal se reconoce que comienza por un pequeño pico de la FSH. Acciones: · Estimula crecimiento folicular en fase preovulatoria · Más estrógenos receptores de LH · Participa en la conversión de folículos primarios a secundarios y a ovulatorio junto con los estrógenos.

LH Menos estable. Vida media de 5 a 210 minutos (corta). Acciones más puntuales. Se segregan por estímulo de LHRH provoca la secreción de un 20 - 30 de FSH y un 70 - 80 de LH: Provoca como mínimo el doble de secreción de LH que de FSH en el mismo flujo de estimulación. Produce rotura folicular. Tiene un pico mesocíclico muy alto (60 - 70 miliunidades /ml) y mantiene tasas más altas en fase preovulatoria (20 - 30 miliunidades/ml). Pico sincronizado con resto de pico. Dura 24 - 72 horas. Se produce 6 horas antes de al ovulación

Morfofisiología del aparato genital femenino .2005.

77.- Agente etiológico del chancro blando: a) Haemophilus ducreyi. b) Calymmatobacterium granulomatosis. c) Neisseria gonorrhoeae. d) Chlamydia trachomatis.

Scout J. (1997) Danforth. Manual de Obstetricia y Ginecología. México. Ed. Mc Grqw Hill Interamericana. Pag 388

El agente etiológico del chancro blando es Haemophilus ducreyi.

78.- ¿A que nos referimos con la fase ebb de la respuesta metabólica al trauma? a) Es la fase final de la respuesta. b) No existe esa fase. d) Es la fase inicial de la respuesta.

e) Es la fase donde se inicia el descenso hormonal de la respuesta.

Schwartz, Principles of Surgery. 7th Edition, McGraw Hill 1999: 3-51 La fase ebb corresponde al inicio de la respuesta metabólica al trauma, sucede en las primeras horas posteriores a la lesión y generalmente se asocia a inestabilidad hemodinámica o a una reducción en el volumen efectivo circulante. En esta fase existe una reducción importante en el gasto energético total y en las pérdidas de nitrógeno urinario. Esta fase se caracteriza por una elevación de catecolaminas y cortisol.

79.- Los análogos de prostaglandinas tienen el siguiente mecanismo de acción: a) b) c) d)

Disminuyen la producción de humor acuoso en pars plana. Disminuyen la producción de humor acuoso en pars alicata. Aumentan la salida de humor acuoso por trabéculo. Aumentan la salida de humor acuoso por la vía uveoescleral.

Los análogos de prostaglandinas son considerados fármacos de primera línea para el glaucoma primario de ángulo abierto logrando su eficacia al facilitar la salida de humor acuoso a través de la vía uveo-escleral. Manual CTO de Medicina y Cirugía. Séptima Edición. McGraw Hill, pgs 1198. Bruce E. Jarrell, R. Anthony Carabasi, Nacional Medical Series for Independent Study. Wiliams & Wilkins, 3rd Edition: 231-264. Los criterior de Ranson al ingreso incluyen, edad mayor a 55 años, leucocitosis mayor a 16,000/mm3, Glucosa mayor a 200mg/dl, DHLmayor a 350 UI/L, TGO mayor a 250 SF unidades %. Los criterios a las 48 horas incluyen Descenso del hematocrito mayor a 10 puntos porcentuales, BUN mayor a 5 mg/dl, Ca menor de 8 µg/dl, Déficit de base mayor a 4 mEq/L

80.- Se trata de paciente masculino de 32 años de edad el cual presenta datos sugestivos de obstrucción intestinal, que medida tomaría para corroborar el diagnóstico:

a) Ecografía abdominal en bipedestación y/o decúbito supino. b) Radiografía simple de abdomen en bipedestación. c) Gammagrafía abdominal. d) TC de abdomen en decúbito lateral o supino.

DATOS RADIOLÓGICOS. La radiología del abdomen es esencial para confirmar el diagnóstico y puede brindar datos acerca de la altura de la obstrucción. Se solicitará radiografía simple de abdomen y en bipedestación o decúbito lateral con rayo horizontal. Intentaremos incluir el diafragma para descartar perforación de víscera hueca.

En la exploración radiológica, suelen descubrirse cantidades anormalmente grandes de gas en intestino y la aparición de niveles hidroaéreos en la placa en bipedestación, producido por el acúmulo de gas y líquido en asas distendidas. Es importante determinar si está distendido el intestino delgado, el colon, o ambos. Las asas de intestino delgado ocupan la porción más central del abdomen, y se disponen 13 transversalmente a modo de peldaños. La imagen de las válvulas conniventes ocupan todo el diámetro del asa (en pila de moneda). Las asas de intestino grueso, se disponen en la periferia, mostrando los pliegues de las austras que no atraviesan completamente el asa. Los pacientes con obstrucción mecánica de intestino delgado no suelen tener gas en colon, y, si existe, es muy escaso. Cuando se observa una dilatación intestinal generalizada de todo el tubo digestivo y heces en ampolla y/o gas distal, debemos sospechar un íleo y dudar del diagnóstico de obstrucción mecánica. En los casos de obstrucción de colon, con válvula ileocecal competente, tendrán distensión de colon, pero poco gas en intestino delgado, cuando la válvula es incompetente, observaremos un patrón radiológico de distensión de intestino delgado y colon. Existen algunas situaciones concretas con imágenes radiológicas características, que pueden orientarnos hacia la etiología de la obstrucción: aerobilia en el caso de ileo biliar o signo del grano de grano de café en el vólvulo. El enema opaco en urgencias es muy útil en caso de obstrucción intestinal de intestino grueso. BIBLIOGRAFÍA. 1) R. Scott Jones M.D. Obstrucción intestinal. En Sabiston, DC: Tratado de Patología Quirúrgica. México DF, Editorial Interamericana, S.A., pp 936-945.

2) McLatchie GR. Oxford. Manual de Cirugía Clínica. Madrid, Ediciones Médicas Folium. 1995. 3) Nyhus LL, Condon RE, Vitello JM. Dolor abdominal. Buenos Aires. Editorial Panamericana S.A. 1995.

View more...

Comments

Copyright ©2017 KUPDF Inc.
SUPPORT KUPDF